Chapter 53: Drug Therapy for Seizure Disorders and Spasticity, Pharmacology Drug Therapy for Seizure Disorders and Skeletal Muscle Disorders, Chapter 53: Drug Therapy for Seizure Disorders and Spasticity, Drug therapy for seizure disorders and spasti…

Réussis tes devoirs et examens dès maintenant avec Quizwiz!

A nurse is writing a plan of care for a client who is taking bethanechol (Urecholine). What would be an appropriate outcome for this client? A. Pupillary dilation B. Increased blood pressure C. Improved bladder function D. Decreased secretions

C. Improved bladder function

A client with a history of malignant hyperthermia is scheduled for surgery. Which agent would the nurse most likely expect to administer?

Dantrolene-Dantrolene is the drug that would be used as prevention and treatment of malignant hyperthermia.

The client is to receive an IV cholinergic medication. When the nurse administers the drug what reaction will the nurse anticipate?

Decrease in heart rate Explanation: Cholinergic drugs decrease, not increase, heart rate. They do not usually have any effect on anxiety.

Which type of seizures involves a loss of consciousness? Partial seizures Generalized seizures Somatosensory seizures Motor seizures

Generalized seizures Explanation: Generalized seizures involve a loss of consciousness.

What type of seizure activity is characterized by generalized tonic-clonic convulsions lasting for several minutes during which the client does not regain consciousness?

status epilepticus

A 44-year-old warehouse worker experienced a painful lower back injury that has not responded to treatment with nonsteroidal anti-inflammatory drugs. Consequently, the man's care provider is considering the short-term use of cyclobenzaprine (Flexeril). Which of the following assessment questions would inform the clinician's decision to prescribe this drug?

"Are you taking antidepressant medications these days?"

The pharmacology instructor is discussing age-related susceptibility to adverse effects of cholinergic drugs. Which statement could the instructor make to accurately describe the influence of age?

"Physiologic changes resulting from the normal aging process place older adults at higher risk for adverse effects." Explanation: Age-related physiologic changes (e.g., to metabolism and excretion) and a higher likelihood of superimposed pathologic conditions (e.g., renal dysfunction) place older adults at increased risk for adverse effects of cholinergic drugs.

A female client's seizure disorder has been successfully controlled by AEDs for years. She and her husband decide that it is time to start a family. She asks the nurse if it is safe for the fetus for her to continue her AEDs as prescribed. What is the nurse's best response? "They are safe during pregnancy." "They are considered teratogenic." "They may interfere with conception." "They are contraindicated during the third trimester."

"They are considered teratogenic." Explanation: Sexually active adolescent girls and women of childbearing potential who require an AED must be evaluated and monitored very closely, because all of the AEDs are considered teratogenic. In general, infants exposed to one AED have a significantly higher risk of birth defects than those who are not exposed, and infants exposed to two or more AEDs have a significantly higher risk than those exposed to one AED.

A client who is experiencing lower back pain has been prescribed cyclobenzaprine. The nurse should provide what health education in order to ensure safe and effective treatment?

"This will likely make you drowsy, so don't take it before doing anything that would require alertness."-Cyclobenzaprine causes drowsiness because of CNS depression. There is no absolute prohibition against using the drug for more than five days; it is metabolized in the liver but is not noted to be highly hepatotoxic. Sedation is an anticipated adverse effect, not a sign of a serious drug reaction.

One week ago, a client began taking ethosuximide 500 mg/day PO for the treatment of absence seizures. The client reports gastrointestinal (GI) upset after taking with the drug. What health education should the nurse provide?

"Try taking your pills at the same time as you eat some food."

The nurse is providing client teaching with a client who is newly diagnosed with epilepsy. The client asks, "Can I still drive to work?" What is the nurse's best response? "You can drive as soon as therapeutic drug levels are established." "Yes, as long as your health care provider agrees and you take your medications regularly." "You'll need to use public transportation because a seizure could occur anytime." "You likely won't be able to until your seizures are controlled by medication"

"You likely won't be able to until your seizures are controlled by medication" Explanation: Clients newly diagnosed with epilepsy will not be able to drive. However, after the client's seizures are controlled (usually for 6 months to 2 years depending on state law), the client may be able to regain the ability to drive. This does not, however, mean that the client has to use public transportation. Serum drug levels are not the determining criterion.

A client with myasthenia gravis has been prescribed dantrolene for the treatment of muscle spasticity. What education should the nurse provide to the client? "You might experience some urinary frequency and urgency. Tell me promptly if it becomes problematic." "If you find you're losing your appetite, let me know so we can create a plan." "Most clients experience relief from their spasticity for 7 to 10 days following the injection." "The care team will follow you closely to ensure that you don't develop a dependence on this medication."

"You might experience some urinary frequency and urgency. Tell me promptly if it becomes problematic." Explanation: Dantrolene often causes urinary frequency and urgency. It is not linked with anorexia and is administered orally each day. Dantrolene is not associated with a high risk for dependence.

A patient with dementia of Alzheimer's disease is administered donepezil HCL. What are the actions associated with cholinesterase inhibitors? A. Increase in the level of acetylcholine in CNS B. Decrease in the level of neurotoxins in the brain C. Increase in the level of cholinesterase in the blood D. Increase in the level of adenosine tri-phosphate in the blood

A. Increase in the level of acetylcholine in CNS

A woman is admitted to the labor and delivery unit with a blood pressure of 200/90 mm Hg. The admitting diagnosis is preeclampsia. The intravenous magnesium sulfate ordered for this patient prevents? A. seizures B. myocardial damage C. tetany D. hypothermia

A. Magnesium sulfate is administered parenterally to prevent seizures.

Which of the following Alzheimer's disease symptoms is treated with cholinesterase inhibitors? A. Mild to moderate dementia B. Severe dementia C. Wandering and aggression D. Irritability

A. Mild to moderate dementia

A nursing student is preparing a care plan and is choosing a nursing diagnosis for her client with Alzheimer's disease. The client is taking Exelon. Which would be an appropriate nursing diagnosis for this client? A. Risk for Injury related to dizziness and fatigue B. Risk for Injury related to being confined to the Alzheimer's unit C. Imbalanced Nutrition: More Than Body Requirements Learning D. Impairment related to decreased hearing

A. Risk for Injury related to dizziness and fatigue

A patient stops taking clonazepam. What adverse effect will occur? A. status epilepticus B. bone marrow depression C. cerebral edema D. lethargy

A. The abrupt withdrawal of clonazepam results in the development of status epilepticus.

An accidental overdose of neostigmine has prompted the emergency administration of atropine. When assessing the patient, the emergency department nurse should take into account that this intervention will not relieve the effects of neostigmine on A) skeletal muscle. B) smooth muscle. C) the heart. D) glands.

Ans: A Feedback: Atropine reverses only the muscarinic effects of cholinergic drugs, primarily in the heart, smooth muscle, and glands. Atropine does not interact with nicotinic receptors and therefore cannot reverse the nicotinic effects of skeletal muscle weakness due to overdose of indirect anticholinergic drugs.

When administering anticholinesterase drugs, what assessment finding would indicate the patient is experiencing a toxic effect of the medication? A) Loss of consciousness B) Abdominal distention C) Hypertension D) Muscle weakness

Ans: D Feedback: The administration of anticholinesterase medications can result in profound muscle weakness. Decreased LOC, abdominal distention, and hypertension are not characteristic adverse effects of the anticholinesterase medications.

The nurse is providing education to a client who has been prescribed an anticholinesterase drug. What caution would the nurse include in the teaching?

Anticholinesterase drugs should be taken with food or milk to decrease the risk of gastric distress and ulceration. Explanation: All anticholinesterase drugs should be taken with food or milk to decrease the risk of gastric distress and ulceration. These drugs are taken at bedtime to avoid the risks associated with dizziness and vertigo, not to prevent nausea. These drugs are not started at very high doses, though doses may be increased after several weeks if initial dosing doesn't achieve the desired effect.

Which would be most important to have readily available for a client who is receiving an indirect-acting cholinergic agonist and develops a severe reaction?

Atropine Explanation: Atropine should be readily available to counteract the severe effects of an indirect-acting cholinergic agonist. Edrophonium is used as the antidote for nondepolarizing neuromuscular junction (NMJ) blockers. Phentolamine is used to as treatment for extravasation of intravenous norepinephrine or dopamine. Naloxone is used to treat narcotic overdose.

The nurse is providing information about how to care for clients during a bioterrorism attack. The nurse knows that which drug will be administered in the case of a sarin attack?

Atropine Explanation: In the case of a nerve gas attack, atropine is the antidote.

A patient is being treated for a seizure disorder with phenytoin (Dilantin). He is admitted to the emergency room with sinus bradycardia. What action will occur regarding his antiepileptic agent? A) Phenytoin (Dilantin) dose will be reduced. B) Phenytoin (Dilantin) will be discontinued. C) Phenytoin (Dilantin) will be given every other day. D) Phenytoin (Dilantin) dose will be increased.

B) Phenytoin (Dilantin) will be discontinued.

A patient has been started on cyclobenzaprine (Flexeril). For the duration of treatment, the nurse should teach the patient to avoid A) OTC vitamin supplements. B) alcohol. C) stool softeners. D) fatty foods.

B) alcohol.

A patient is taking carisoprodol for back spasms related to an occupational injury. Which of the following is most important to teach the patient? A. to take the medication at 8:00 am, 2:00 pm, and 8:00 pm B. to know the signs and symptoms of an idiosyncratic reaction C. to take the medication between meals D. to stop the medication with the first sign of abdominal cramping

B. A patient who takes carisoprodol should receive instruction about the signs and symptoms of an idiosyncratic reaction.

A nurse is caring for a 49-year-old client who has been taking bethanechol for treatment of neurogenic bladder. After one week, the client develops sweating, heavy salivation, and bronchospasm. What would the nurse suspect is happening with this client? A. Myasthenic crisis B. Cholinergic crisis C. Anaphylactic reaction D. Pulmonary edema

B. Cholinergic crisis

Which medication would the nurse expect to administer if prescribed to achieve skeletal muscle relaxation? Baclofen Allopurinol Alendronate Hydroxychloroquine

Baclofen Baclofen is an example of a skeletal muscle relaxant. Allopurinol would be administered to treat gout. Alendronate would be administered to treat osteoporosis. Hydroxychloroquine would be used to treat rheumatoid arthritis.

The nurse has been educating the patient on the self-administration of phenytoin (Dilantin). Which of the following statements by the patient demonstrates an understanding of the medication? A) "I'll only take the drug when I feel an impending seizure." B) "I'll reduce my dose if I remain seizure free." C) "I'll make sure to take the drug with food." D) "I'll stop taking the drug if I don't have a seizure for 8 weeks."

C) "I'll make sure to take the drug with food."

A patient with cortical focal seizures has been prescribed phenobarbital. What adverse reaction should the nurse monitor for in the patient? Gingival hyperplasia Urticaria Ataxia CNS depression

CNS depression Explanation: The nurse should monitor CNS depression in the patient undergoing phenobarbital treatment. Gingival hyperplasia is an adverse reaction in a patient administered ethotoins. Ataxia and urticaria are adverse reactions in patients undergoing anticonvulsant ethosuximide therapy.

A student asks the pharmacology instructor to describe the function of a cholinergic agonist. What would the instructor reply?

Cholinergic agonists increase the activity of acetylcholine receptor sites throughout the body. Explanation: Cholinergic agonists are drugs that increase the activity of acetylcholine receptor sites throughout the body. Dopamine, GABA, and norepinephrine are not associated with cholinergic agonist function.

A client in the clinic with a history of epilepsy has just tested positive for pregnancy. What is the nurse's next action? Consult with the primary health care provider. Discuss the most serious birth defects that may occur due to treating epilepsy during pregnancy. Explain how to taper off her seizure medication as the risk for seizures decreases during pregnancy. Advise the client to continue her current medication for epilepsy.

Consult with the primary health care provider. Explanation: The nurse should first consult with the primary health care provider. Research suggests an association between (a) the use of anticonvulsants by pregnant women with epilepsy and (b) an increased incidence of birth defects. The use of anticonvulsants is not discontinued in pregnant women with a history of major seizures because of the danger of precipitating status epilepticus. However, when seizure activity poses no serious threat to the pregnant woman, the primary health care provider may consider discontinuing use of the drug during pregnancy.

A nurse is assessing a client who has come to the emergency department reporting back spasms. The client states a history of opioid addiction and does not want to take any drug that "puts me at risk of becoming physically dependent." Which medication would the nurse question, if ordered? Methocarbamol (Robaxin) Orphenadrine (Norflex) Cyclobenzaprine (Flexeril) Metaxalone (Skelaxin)

Cyclobenzaprine (Flexeril) Explanation: The nurse would be concerned if the health care provider ordered cyclobenzaprine because long-term use of this drug may result in physical dependence. The other drugs listed are not associated with physical dependence.

The client is taking cyclobenzaprine for muscle spasms secondary to an injury to the lumbar spine that occurred while lifting a motor at work. The client is being seen for a follow-up visit by the health care provider. The client reports dry mouth, blurred vision, and constipation. Why is the client having these side effects from cyclobenzaprine?

Cyclobenzaprine produces an anticholinergic response-Cyclobenzaprine relieves muscle spasms through a central action, possibly at the level of the brain stem, with no direct action on the neuromuscular junction or the muscle involved. The common adverse effects of cyclobenzaprine are related to its CNS depression and anticholinergic activity. The most common adverse effects are drowsiness, dizziness, and dry mouth.

A nurse administers baclofen (Lioresal) to a young man for a back injury. Which of the following interventions is most important related to the administration of baclofen? A. Assess heart rate B. Assess for edema C. Assess blood sugar D. Assess for increased spasticity

D. Assess for increased spasticity

An 18-year-old man is having an open reduction external fixation of a hip following a football injury. During surgery, he develops a fever of 105°F. Which of the following medications is administered? A. cyclobenzaprine hydrochloride B. dexamethasone sodium phosphate C. phenytoin D. dantrolene sodium

D. Dantrolene is the skeletal muscle of choice for the treatment of malignant hyperthermia.

A male infant is admitted to the emergency department. His parents report that he had a seizure. The infant's aural temperature is 104.3°F. What is the most likely cause of the seizure? A. head injury B. developmental delay C. sepsis D. fever

D. The cause of the infant's seizure is his fever. Tis is a febrile seizure.

A nurse administers carisoprodol to a client for the treatment of an acute musculoskeletal condition. The nurse would be alert for which adverse effect after administering the drug?

Drowsiness

A client has been prescribed carbamazepine for the prevention of seizures. What action should the nurse perform? Educate the client about the need to take the pills as scheduled Educate the client about taking the medication at the first sign of impending seizure activity Establish intravenous access Teach the client how to self-administer subcutaneous injections

Educate the client about the need to take the pills as scheduled Carbamazepine is administered orally on an ongoing basis. It is not an emergency treatment for seizure activity.

A client is receiving a hydantoin as treatment for tonic-clonic seizures. The nurse includes a discussion of what when teaching the client about this drug? Withdrawal syndrome Possible leukocytosis Physical dependence Gingival hyperplasia

Gingival hyperplasia Explanation: Hydantoins may cause gingival hyperplasia, severe liver toxicity, and bone marrow suppression. Physical dependence and withdrawal syndrome are associated with benzodiazepine

Signs and symptoms of what would necessitate discontinuation of dantrolene therapy?

Hepatic dysfunction-Dantrolene therapy must be discontinued at any sign of liver dysfunction. Intermittent GI upset, visual disturbances, and urinary retention are associated adverse effects of the drug and, although problematic, do not necessitate discontinuing the drug.

A patient with dementia of Alzheimer's disease is administered donepezil HCL. What are the actions associated with cholinesterase inhibitors?

Increase in the level of acetylcholine in CNS Explanation: Administration of cholinesterase inhibitors results in an increase in the level of acetylcholine in CNS. Administration of cholinesterase inhibitors does not decrease the neurotoxins in the brain or increase the level of adenosine tri-phosphate in the blood. Cholinesterase inhibitors do not increase the level of cholinesterase in the blood.

A female patient has been administered donepezil HCL for dementia. The patient has informed the nurse that she has also been taking nonsteroidal anti-inflammatory drugs. Which interaction should the nurse monitor for in this patient?

Increased risk of GI bleeding Explanation: The interaction of nonsteroidal anti-inflammatory drugs with cholinesterase inhibitors causes increased risk of GI bleeding, which should be monitored for. Interaction of anticholinergics with cholinesterase inhibitors causes decreased effectiveness of anticholinergics. Interaction of theophylline with cholinesterase inhibitors causes increased risk of theophylline toxicity. Interaction of nonsteroidal anti-inflammatory drugs with cholinesterase inhibitors does not decrease the GI absorption of the drug.

When describing the action of zonisamide, which would the nurse include? Depression of motor nerve output Decrease in conduction through nerve pathways Depression of the cerebral cortex Inhibition of sodium and calcium channels

Inhibition of sodium and calcium channels Explanation: Zonisamide inhibits voltage-sensitive sodium and calcium channels, thus stabilizing the nerve cell membranes and modulating calcium-dependent presynaptic release of excitatory neurotransmitters. Hydantoins decrease the conduction through nerve pathways. Barbiturates and barbiturate-like agents depress the cerebral cortex and motor nerve output.

A client comes to the emergency department. The client is experiencing continuous seizure activity without any interruptions. Which anticonvulsant would the nurse anticipate that the primary health care provider would most likely prescribe initially? Phenytoin Valproic acid Lorazepam Fosphenytoin Clorazepate

Lorazepam Explanation: Lorazepam (Ativan) is the drug of choice for this condition. However, because the effects of lorazepam last less than 1 hour, longer-lasting anticonvulsants such as phenytoin or fosphenytoin may be given to continue to control the seizure activity. Valproic acid is used to treat epilepsy, migraine headache, and mania. Clorazepate is used to treat focal seizures, anxiety disorders, and alcohol withdrawal.

An older adult is prescribed a skeletal muscle relaxant for reports of neck pain. What is the top priority of care for the nurse to teach the family? Administer the medication with meals. Monitor the client before ambulating. Instruct on how to assess client's pain level. Give the medication at the same time each day.

Monitor the client before ambulating. Explanation: Safety is the top priority concern, because skeletal muscle relaxants can cause drowsiness. Administering the medication with meals can reduce GI distress but is not the top priority safety concern. Checking the client's pain level is important but not the top priority safety concern. Giving the medication at the same time each day may help the caregivers remember it, but is not the top priority safety concern.

A client with advanced-stage Alzheimer's disease is seen by a health care provider who prescribes a newer drug that is not a cholinesterase inhibitor. This drug is a N-methyl-D-asparate (NMDA) receptor antagonist. This drug is better known as:

Namenda. Explanation: Cholinesterase inhibitors are not used in late-stage Alzheimer's disease. A newer group of drugs, NMDA receptor antagonists, is available. Namenda is thought to work by decreasing the excitability of neurotransmission caused by an excess of the amino acid glutamate in the CNS.

The nurse examines reviews the medical administration record and notes the medication lamotrigine needs to be administered. The client has developed a red scaly rash with fluid-filled blisters and purpuric areas all over the client's body. Which action(s) should the nurse take? Select all that apply. Notify the primary care provider immediately. Assess vital signs. Administer the dose of lamotrigine. Document the skin assessment. Cleanse affected areas with soap and water.

Notify the primary care provider immediately. Assess vital signs. Document the skin assessment.

A nurse is administering lorazepam to a client with status epilepticus. The nurse will be prepared to administer which additional drug to treat the status epilepticus for the next several hours? Ethosuximide Phenytoin Ethotoin Zonisamide

Phenytoin Explanation: Due to the short effects of lorazepam, a longer-acting anticonvulsant, such as phenytoin, is given to continue control of seizure activity. Ethotoin is a hydantoin that is used to treat tonic-clonic seizures. Ethosuximide is a succinimide that is used to treat focal seizures. Zonisamide is a nonspecified preparation and used to treat focal seizures as well.

A client with a spinal cord injury has developed central spasticity and the care provider wishes to administer a muscle relaxant intrathecally. What is the nurse's best action? Prepare to administer baclofen. Contact the care provider to question the treatment plan. Ensure that the client has patent venous access. Prepare to administer carisoprodol.

Prepare to administer baclofen. Explanation: Baclofen is available in oral and intrathecal forms and can be administered via a delivery pump for the treatment of central spasticity. Soma cannot be delivered in this manner and there is no obvious reason to question such an order. The client likely needs venous access for other fluids and medications, but it is not necessary for intrathecal administration.

The nurse has completed a teaching with a client prescribed phenytoin daily for seizure activity. The nurse determines the session is successful after the client correctly chooses which potential adverse reaction that can occur if the client misses a dose of medication? CNS depression Hypotension Recurrence of seizures Nystagmus

Recurrence of seizures Explanation: Recurrence of seizure activity may result from abrupt discontinuation of the drug, even when the anticonvulsant is being administered in small daily doses. Abrupt discontinuation of the drug does not cause CNS depression, hypotension, or nystagmus. CNS depression, hypotension, and nystagmus are adverse reactions of phenytoin.

When reviewing a client's history, which condition would the nurse identify as contraindicating the use of a centrally acting skeletal muscle relaxant? Rheumatic disorder Depression Epilepsy Hypertension

Rheumatic disorder Explanation: Centrally acting skeletal muscle relaxants would be contraindicated for treatment of muscle spasms related to a rheumatic disorder. Epilepsy, depression, or hypertension would not necessitate cautious use.

A client with a history of partial seizures has been taking lamotrigine for the past several days. The client calls the clinic and reports the development of a facial and torso rash to the nurse. What is the nurse's best action?

Tell the client to take no further doses and come be assessed at the clinic immediately.-The nurse should inform the client to discontinue the drug and return to the clinic. Rashes associated with the use of lamotrigine can be life-threatening. The client needs to return to the clinic to be evaluated and will need a change of medication. Recommending another medication is insufficient, and is also beyond the nurse's scope. High-fat foods are of no benefit.

A client has been prescribed topical cholinergic drug therapy for treatment of glaucoma. The nurse would inform the client that which effect might occur when beginning therapy?

Temporary decrease in visual sharpness and headache Explanation: The nurse should inform the client that a temporary reduction of visual acuity (sharpness) and headache may occur at the beginning of the therapy. Nausea, increased salivation, abdominal cramping and diarrhea, and flushing of the skin are adverse reactions associated with cholinergic drugs not administered topically.

The nursing instructor is teaching a student about the drugs used for Alzheimer's disease. Even though cholinesterase inhibitors do not cure the disease, the instructor informs the student that they do help to slow the progression. The instructor then asks the student, "When a drug is stopped due to side effects, what happens to the client?" The student's best response would be:

The client loses any benefit they have received from the drug. Explanation: When a cholinesterase inhibitor has to be stopped due to adverse effects then within 6 weeks of the discontinuation of the therapy, the client will lose any benefit they have received from the drugs.

A 60 year-old female client has multiple sclerosis accompanied by muscle spasticity. The client has responded well to dantrolene 200 mg PO daily in divided doses. What assessment finding should the nurse prioritize for communication to the client's provider?

The client's most recent laboratory results show an upward trend in AST and ALT levels

A client's health care provider has prescribed baclofen in an effort to treat neuropathic cancer pain. What education should the nurse prioritize when teaching the client about this new medication? The importance of regularly scheduled liver function testing The need to maintain a diet that is high in iron and folic acid The importance of ensuring safety related to possible sedation The importance of maintaining a sterile central venous catheter

The importance of ensuring safety related to possible sedation Explanation: Baclofen carries a risk of CNS depression and a consequent threat to safety. The use of baclofen does not necessitate regular liver function testing or dietary changes. Baclofen is administered orally or intrathecally, not intravenously.

A nurse is to administer an anticonvulsant drug. The nurse understands that this drug is classified as an oxazolidinedione. Which drug would the nurse most likely be preparing to administer? Diazepam Trimethadione Gabapentin Levetiracetam

Trimethadione Explanation: Trimethadione is classified as an oxazolidinedione. Diazepam is classified as a benzodiazepine. Gabapentin is classified as a miscellaneous anticonvulsant. Levetiracetam is classified as a miscellaneous anticonvulsant.

The nurse is providing information about how to care for clients during a bioterrorism attack. The nurse knows that which drug will be administered in the case of a sarin attack? a. Atropine b. Neostigmine c. Donepezil d. Bethanechol

a. Atropine In the case of a nerve gas attack, atropine is the antidote.

A client has been experiencing increased muscle spasticity since being diagnosed with multiple sclerosis. What drug is most likely to meet this client's needs? orphenadrine metaxalone baclofen cyclobenzaprine

baclofen Explanation: Baclofen is used for treatment of muscle spasticity associated with neuromuscular diseases such as multiple sclerosis. Cyclobenzaprine, metaxalone, and orphenadrine are used for relief of discomfort associated with painful, acute musculoskeletal conditions.

Which medication would the nurse expect to administer orally once a day to a client with AD? a. Rivastigmine b. Memantine c. Donepezil d. Galantamine

c. Donepezil Donepezil is given once a day. While rivastigmine, galantamine, and memantine are given twice a day.

The nurse is caring for a client who has a disorder of muscle contraction and relaxation. The nurse should prioritize the assessment of what laboratory value? magnesium chloride potassium calcium

calcium Explanation: Calcium is released from the sarcoplasmic reticulum, which leads to the binding of calcium with troponin-tropomyosin. This leads to contraction of the muscle fiber. The calcium pump then moves calcium back into the sarcoplasmic reticulum, which leads to relaxation of muscle fiber. Chloride, magnesium, and potassium are not involved in this proces

A client with hypertension is to receive an anticholinergic agent. The nurse would be especially alert for: a. bladder obstruction. b. paralytic ileus. c. increased intraocular pressure. d. increased blood pressure.

d. increased blood pressure. A client with hypertension who receives an anticholinergic is at risk for additive hypertensive effects due to the dominance of the sympathetic system with parasympathetic blockage. Bladder obstruction, paralytic ileus, and increased intraocular pressure are contraindications for the use of an anticholinergic agent.

A client whose Parkinson's disease is being treated with tolcapone should concurrently take what drug? a. benztropine b. ipratroprium c. atropine d. levodopa/carbidopa

d. levodopa/carbidopa

A surgical client has developed malignant hyperthermia. Which medication can be used to treat this health emergency?

dantrolene

The nurse is caring for a client who is receiving neostigmine for the treatment of myasthenia gravis. The nurse is aware that a potential complication of the treatment is cholinergic crisis, which is characterized by which event?

excessive stimulation of the parasympathetic nervous system Explanation: A cholinergic crisis is characterized by excessive stimulation of the parasympathetic nervous system. If early symptoms are not treated, hypotension and respiratory failure may occur. At high doses, anticholinesterase drugs weaken, rather than strengthen, skeletal muscle contraction because excessive amounts of acetylcholine accumulate at motor endplates and reduce nerve impulse transmission to muscle tissue.

A nurse is working with a 39-year-old client who is experiencing muscle spasms associated with multiple sclerosis. The client reports needing help with basic care The client's health care provider has prescribed baclofen (Lioresal). The nurse and the client agree that an appropriate goal related to helping to maintain self-worth would be to: experience relief of pain. experience improved motor function. increase self-care in activities of daily living. take medication independently.

increase self-care in activities of daily living. Explanation: All of the options are goals that the nurse would like the client to achieve. However, the goal related to self-worth would be to increase self-care in activities of daily living. Loss of ability to dress oneself puts the client at risk for decreased self-esteem and feelings of low self-worth.

The nurse is caring for an 84-year-old client in the acute care facility who was newly diagnosed with a seizure disorder. Before starting the client on an antiepileptic medication that will be continued after discharge, what laboratory studies should the nurse assess? Select all that apply. serum drug levels liver function studies renal function studies cardiovascular function studies central nervous system function studies

liver function studies renal function studies Baseline kidney and liver function tests should be done and results will guide dosing of the antiepileptic medication because clients with liver or kidney disease will require lower dosages of medication. Serum drug levels will not be drawn until after specific medications are started. Cardiovascular and central nervous system function studies are not indicated.

A client has been diagnosed with cerebral palsy accompanied by muscle spasticity. The nurse should identify what causative factor of this client's symptoms? imbalances between neurotransmitter levels and the levels of enzymes that modulate their reuptake inherited defects of the peripheral nervous system irreversible injury to muscle tissue nerve damage within the central nervous system

nerve damage within the central nervous system Explanation: Muscle spasticity is the result of damage to neurons within the central nervous system (CNS) rather than injury to peripheral structures such as the musculoskeletal system. Serotonin is not involved in the process of muscle contraction and relaxation. The etiology of spasticity is not rooted in neurotransmitter disruptions, even though these may subsequently be affected.

What is the most common drug of choice prescribed for the treatment of seizure activity in adults?

phenytoin-The oldest and most widely used antiepileptic drug (AED), phenytoin is often the initial drug of choice, especially in adults. None of the remaining drugs are as widely prescribed.

An adolescent taking oral contraceptives has been prescribed an anticonvulsant medication. The nurse should tell the client to do which? use another form of birth control, such as condoms. watch for signs of hyperglycemia, such as increased thirst, hunger, and urination. be aware that these two drugs will interact and increase seizure activity initially. stop driving due to the increased depressant effects and excessive drowsiness.

use another form of birth control, such as condoms. Explanation: Anticonvulsants and oral contraceptives interact, leading to decreased effectiveness of birth control. This can result in breakthrough bleeding or unintended pregnancy. Anticonvulsants and antidiabetic medications interact, resulting in increased blood glucose levels. Anticonvulsants and antiseizure medications taken together may increase seizure activity. Anticonvulsants interact with analgesics and alcohol, not oral contraceptives, to cause increased depressant effects.

A client is taking rivastigmine for Alzheimer disease. The nurse will evaluate the client regularly for:

weight loss. Explanation: A client taking rivastigmine is at risk for substantial gastrointestinal adverse reactions, including nausea and vomiting, anorexia, and weight loss. It would be important for the nurse to monitor the client's weight and plan accordingly. An increase or decrease in blood pressure is not a concern with this drug.

The community health nurse also assists the health care provider in the decision-making process for AED drug titration by performing which function? Counting the number of pills in the AED drug bottle to ensure compliance Administering the AED medication to the client Ensuring that the client makes the appointments for serum drug levels Monitoring the client for signs and symptoms of seizure activity during the monthly visit

Ensuring that the client makes the appointments for serum drug levels Explanation: The community health nurse may assist the provider to titrate drug doses by ensuring that the client keeps appointments for serum drug level testing and follow-up care

After teaching a class on drug classes used to treat seizures, the instructor determines that the teaching has been successful when the students identify which drug as most commonly used in the treatment of absence seizures? Mephobarbital Ethotoin Ethosuximide Primidone

Ethosuximide Explanation: Ethosuximide is most frequently used to treat absence seizures. Mephobarbital, ethotoin, and primidone are typically used for tonic-clonic seizures

An older adult resident of a long-term care facility is experiencing muscle spasticity and has just been prescribed a centrally acting skeletal muscle relaxant. The client has comorbidities of early stage Alzheimer's disease and chronic obstructive pulmonary disease (COPD). What nursing action should the nurse prioritize for adding to the client's care plan? Falls prevention measures Seizure precautions Continuous pulse oximetry Deep breathing and coughing exercises

Falls prevention measures Explanation: The client's combination of CNS depression from the muscle relaxant and an underlying cognitive deficit creates a risk for falls. This combination is not associated with seizures and will not likely exacerbate the client's COPD; respiratory assessments and interventions would not likely need to be amended.

A 44-year-old warehouse worker experienced a painful lower back injury that has not responded to treatment with nonsteroidal anti-inflammatory drugs. Consequently, the man's care provider is considering the short-term use of cyclobenzaprine (Flexeril). Which of the following assessment questions would inform the clinician's decision to prescribe this drug? "Are you taking antidepressant medications these days?" "Do you have a history of eczema or psoriasis?" "How would describe your activity level since your injury?" "Have you ever been told that you have an enlarged prostate gland?"

"Are you taking antidepressant medications these days?" Explanation: Because of its similarity to the TCAs, cyclobenzaprine is also contraindicated for use within 14 days of administration of monoamine oxidase inhibitors (MAOIs). Prostatic enlargement, skin disorders, and low activity level do not preclude the use of cyclobenzaprine.

A client who has been prescribed bethanechol asks a nurse to explain how the drug works. What would be a correct response?

"Bethanechol is a direct-acting cholinergic drug that causes contraction of the smooth muscles of the bladder and passage of urine." Explanation: Bethanechol is a direct-acting cholinergic drug that causes contraction of the smooth muscles of the bladder and passage of urine. Bethanechol is not an indirect acting cholinergic drug nor is it a diuretic. It does not increase water production in the kidneys; kidneys do not produce water.

An 11 year-old client has been diagnosed with epilepsy and prescribed phenytoin 100 mg PO b.i.d. What statement by the client's parent suggests an accurate understanding of the client's medication regimen? "I will make sure my child takes the medication on an empty stomach." "I will stop the drug immediately if any side effects occur." "I will make sure my child has routine visits to the dentist." "I will weigh my child daily and feed them a high-calorie diet."

"I will make sure my child has routine visits to the dentist." Explanation: Gingival hyperplasia is common in clients, especially children, who take phenytoin, which makes regular dentist visits important to oral health. Taking the medication on a full stomach or with meals reduces gastrointestinal (GI) adverse effects. The mother should call the healthcare provider if adverse effects are noted and needs to understand the risks associated with abrupt withdrawal of the medication. Daily weight taking and high-calorie diets are not necessary during phenytoin administration.

A 50-year-old woman has begun taking baclofen to treat her recently diagnosed multiple sclerosis (MS). What teaching point should the nurse provide to the patient about her new drug regimen?

"Make sure that you don't stop taking baclofen suddenly because it might cause your symptoms to rebound quite sharply."-Abrupt discontinuation of baclofen should be avoided, as it may result in severe side effects such as confusion, seizures, exacerbations of severe spasticity, hallucinations, and other psychiatric disorders.

A 50-year-old woman has begun taking baclofen to treat her recently diagnosed multiple sclerosis (MS). What teaching point should the nurse provide to the patient about her new drug regimen?

"Make sure that you don't stop taking baclofen suddenly because it might cause your symptoms to rebound quite sharply."-Abrupt discontinuation of baclofen should be avoided, as it may result in severe side effects such as confusion, seizures, exacerbations of severe spasticity, hallucinations, and other psychiatric disorders. Baclofen is not available over the counter and is prescribed on a scheduled basis, rather than in response to acute symptoms. It is not necessary to stop taking other drugs prior to or during treatment with baclofen.

A client with myoclonic seizures has been prescribed clonazepam as an adjunctive treatment by the neurologist. What teaching should the nurse prioritize when explaining this new drug regimen to the client? "You'll have to come to the hospital's outpatient clinic twice a week to have this administered intravenously." "We will need to monitor your blood sugar quite closely while you're taking this drug." "Make sure you don't stop taking this abruptly because that might bring on a seizure." "You might find that you'll feel somewhat agitated for the first few days that you take this drug."

"Make sure you don't stop taking this abruptly because that might bring on a seizure." Explanation: Tolerance to the antiseizure effects of clonazepam can occur within months. Abrupt withdrawal of the drug induces seizures, including status epilepticus. Clonazepam causes drowsiness, not agitation, and it is unnecessary to monitor blood sugar closely. Administration is oral, not intravenous.

One week ago, a client began taking ethosuximide 500 mg/day PO for the treatment of absence seizures. The client reports gastrointestinal (GI) upset after taking with the drug. What health education should the nurse provide? "Take the drug 1 hour before or 2 hours after a meal to minimize stomach upset." "Taking over-the-counter antacids before and after the dose helps for some clients." "Try taking your pills at the same time as you eat some food." "Contact your health care provider to see if there's an alternative drug that would work for you."

"Try taking your pills at the same time as you eat some food." If GI irritation occurs with ethosuximide or other anticonvulsants, the client should be encouraged to take the medication with food to reduce this adverse effect. Taking the drug 1 to 2 hours after meals would not reduce this effect. The nurse must be cautious when recommending the use of OTC medications. GI effects are unlikely to necessitate a change in medications.

A 43-year-old woman was diagnosed with multiple sclerosis 2 years ago and has experienced a recent exacerbation of her symptoms, including muscle spasticity. Consequently, she has been prescribed Dantrolene (Dantrium). In light of this new addition to her drug regimen, what teaching point should the woman's nurse provide? "You might find that this drug exacerbates some of your muscle weakness while it relieves your spasticity." "We'll need to closely monitor your blood sugar levels for the next week." "This will likely relieve your muscle spasms but you'll probably develop a certain amount of dependence on the drug over time." "There's a small risk that you might experience some hallucinations in the first few days that you begin taking this drug."

"You might find that this drug exacerbates some of your muscle weakness while it relieves your spasticity." Explanation: Dantrolene causes weakness because of its generalized reduction of muscle contraction. It is not associated with drug dependence, hyperglycemia, hypoglycemia, or hallucinations.

The nurse knows the importance of doing an ongoing assessment on clients when they are taking the cholinesterase inhibitors. This ongoing assessment should include which? Check all that apply.

-Cognitive abilities -Functional abilities Explanation: Ongoing assessments of clients taking the cholinesterase inhibitors include both mental and physical assessments. Cognitive and functional abilities are assessed. Initial assessments will be compared to the ongoing assessments to monitor the client's improvement (if any) after taking the drugs. The other options do not tell us about the effects of the medications.

A 12-year-old boy comes to the school nurse's office after falling during gym class. The nurse assesses the child and notes that he has bruising and petechiae over most of his legs, arms, and torso. The child has a history of absence seizures that are treated with valproate. Based on the child's history, what would the nurse suspect? A. abuse B. leukemia C. adverse effect D. anemia

. C. Valproic acid has an adverse effect of bleeding. The first sign of bleeding in this client is bruising and petechiae.

A client is receiving baclofen at 8 AM. The nurse would monitor the client for evidence of maximum effect at which time? 9 AM 10 AM 11 AM 12 PM

10 AM Explanation: Baclofen peaks in 2 hours after administration, so maximum effectiveness would be noted at this time, which in this case would be 10 AM.

A client who is receiving phenytoin has a serum drug level drawn. Which result would the nurse interpret as within the therapeutic range? 4 mcg/mL 12 mcg/mL 30 mcg/mL 22 mcg/m

12 mcg/mL Explanation: The therapeutic serum phenytoin levels range from 10 to 20 mcg/mL. Thus, a level of 12 mcg/mL would fall within this range

A client who is receiving phenytoin has a serum drug level drawn. Which result would the nurse interpret as within the therapeutic range? 30 mcg/mL 12 mcg/mL 4 mcg/mL 22 mcg/mL

12 mcg/mL Explanation: The therapeutic serum phenytoin levels range from 10 to 20 mcg/mL. Thus, a level of 12 mcg/mL would fall within this range.

A 36-year-old man has been taking extended-release carbamazepine for 1 year. He is in the clinic every 3 months for follow-up visits, which include a drug blood level. Which of the following carbamazepine blood levels would indicate a potential risk for adverse effects? 2 mcg/mL 6 mcg/mL 10 mcg/mL 15 mcg/mL

15 mcg/mL Explanation: The therapeutic blood level for the drug ranges from 4 to 12 mcg/mL for an adult. A level of 15 mcg/mL is higher than the therapeutic level and could cause increased adverse effects or possible toxic effects.

The nurse is monitoring the serum carbamazepine level of a client. Which result would lead the nurse to notify the prescriber that the client most likely needs an increased dosage? 6 mcg/mL 4 mcg/mL 2 mcg/mL 8 mcg/mL

2 mcg/mL Explanation: Therapeutic serum carbamazepine levels range from 4 to 12 mcg/mL. Therefore, a level under 4 mcg/mL would suggest that the drug has not reached therapeutic levels, so the dosage may need to be increased.

The nurse is monitoring the serum carbamazepine level of a client. Which result would lead the nurse to notify the prescriber that the client most likely needs an increased dosage? 2 mcg/mL 4 mcg/mL 8 mcg/mL 6 mcg/mL

2 mcg/mL Explanation: Therapeutic serum carbamazepine levels range from 4 to 12 mcg/mL. Therefore, a level under 4 mcg/mL would suggest that the drug has not reached therapeutic levels, so the dosage may need to be increased.

The primary health care provider prescribes diazepam 10 mg IV to be administered to a client to control their seizures. The nurse would administer this drug over which time frame? 1 min 2 min 5 min 10 min

2 min Explanation: When used to control seizures, diazepam is administered IV pushed slowly as close as possible to the IV site, allowing at least 1 min for each 5 mg of drug. For a dosage of 10 mg, the nurse would administer the drug over 2 min.

The perioperative nurse is caring for a client who requires an umbilical hernia repair but who has a known family history of malignant hyperthermia. The client has been prescribed dantrolene 2.5 mg/kg IV one hour before surgery. The client weighs 121 lbs. The nurse reconstitutes a single-use vial as per the manufacture's instructions, yielding a solution with a concentration of 50 mg/mL. How many mL of the reconstituted dantrolene solution should the nurse add to the client's bag of intravenous fluid? 6.87 mL 2.5 mL 6.05 mL 2.75 mL

2.75 mL Explanation: The client's weight in kilograms is 55 kg (121 lbs divided by 2.2). The ordered dose is 2.5 mg/kg and 55 kg x 2.5 mg = 137.5 mg. There are 50 mg in each mL of the reconstituted solution. Dividing the dose required (137.5 mg) by the dose available (50 mg/mL) yields 2.75 mL

A client is receiving phenytoin. The nurse monitors the client's plasma drug level. Which level would alert the nurse to the possibility of toxicity? 27 µg/mL 10 µg/mL 15 µg/mL 18 µg/mL

27 µg/mL Explanation: Phenytoin plasma levels between 10 and 20 µg/mL give optimal anticonvulsant effect. However, many clients achieve seizure control at lower serum concentration levels. Levels greater than 20 µg/mL are associated with toxicity.

A nurse caring for a client receiving bethanechol for urinary retention should advise the client that voiding usually occurs how long after oral administration?

30 to 90 minutes Explanation: A nurse caring for a client receiving bethanechol (Urecholine) for urinary retention should advise the client that voiding usually occurs 30 to 90 minutes after an oral dose is administered. The time frame of 5 to 15 minutes does not allow enough time for the medication to be absorbed and distributed. The time frame of 2 to 4 hours indicates that the medication will not work and the time frame of 12 to 24 hours indicates that the client will have bladder distention and that an insertion of a catheter may need to be placed to relieve the pressure exerted on the bladder.

A nurse is aware that baclofen (Lioresal) is a centrally-acting spasmolytic that has been demonstrated to be safe and effective in the treatment of numerous disorders of muscle spasm and spasticity. The nurse should question the order if baclofen is prescribed for which of the following patients? A 22-year-old man whose muscle spasticity is attributable to a spinal cord injury that he suffered while snowboarding. A 49-year-old woman whose recent motor disturbances have culminated in a diagnosis of multiple sclerosis. A 14-year-old boy who has a diagnosis of cerebral palsy and who has experienced a recent increase in muscle spasticity. A 70-year-old man whose Parkinson disease has worsened in recent months, resulting in decreased mobility and self-care.

A 70-year-old man whose Parkinson disease has worsened in recent months, resulting in decreased mobility and self-care. Explanation: Baclofen is useful in the treatment of spasticity and myoclonus resulting from disorders such as MS, cerebral palsy, and traumatic injury to the spinal cord. However, it is not useful in treating spasms that follow a CVA or stroke, or those that occur in Parkinson disease or Huntington chorea.

A nurse who gives care on a neurological floor is working with several clients. Which client should the nurse prioritize for further assessment and possible interventions? A client receiving pregabalin who is not responsive to verbal stimuli A client being treated with phenobarbital whose blood pressure is 106/69 mmHg A client who will be discharged on carbamazepine and who requires education A client who is anxious about undergoing electroencephalography to investigate a recent absence seizure

A client receiving pregabalin who is not responsive to verbal stimuli Explanation: A client's decreased level of consciousness would be a priority for further assessment. It could be indicative of excessive CNS depression if the client is unable to be roused by voice. The client with a blood pressure of 106/69 may require further monitoring and possible interventions, but this blood pressure is not so low as to be considered an emergency. Addressing clients' learning needs and anxiety is also important, but less time dependent than a client who is not rousable.

A nurse who gives care on a neurological floor is working with several clients. Which client should the nurse prioritize for further assessment and possible interventions?

A client receiving pregabalin who is not responsive to verbal stimuli-A client's decreased level of consciousness would be a priority for further assessment. It could be indicative of excessive CNS depression if the client is unable to be roused by voice.

The nurse is assessing the records of a client in preparation to administer an antiepileptic and notes the new order for periodic complete blood counts. Which new medication(s) does the nurse prepare to administer? Select all that apply.

A client taking carbamazepine or felbamate should be monitored closely for pancytopenia. Clients receiving phenytoin should have routine drug serum levels evaluated to monitor for possible toxicity. Valproic acid and zonisamide do not require a specific blood evaluation.

The neurological nurse cares for several clients who have seizure disorders. Which client should the nurse monitor most closely for indications of drug dependence? A client with a history of myoclonic seizures who takes valproic acid A client with a history of tonic-clonic seizures who takes phenobarbital A client receiving IV phenytoin to prevent seizures post-neurosurgery A client receiving ethosuximide for the prevention of absence seizures

A client with a history of tonic-clonic seizures who takes phenobarbital Explanation: Barbiturates, such as phenobarbital, have an associated risk for dependence. This is not true of the other listed drugs.

A nurse is teaching a client with gout the importance of getting adequate fluids. The nurse determines the session is successful when the client correctly plans to drink at least how much fluid each day? 1000 mL 1500 mL 2000 mL 3000 mL

A client with gout is encouraged to drink at least 3000 mL of fluid per day to promote uric acid excretion.

A patient with multiple sclerosis is admitted to the medical division for treatment of severe spasticity. What medication is used to treat spasticity and is administered intrathecally? A) Baclofen (Lioresal) B) Carisoprodol (Soma) C) Diazepam (Valium) D) Dantrolene (Dantrium)

A) Baclofen (Lioresal)

A patient is recovering from a stroke and has developed severe muscle contractions. Which of the following medications will inhibit the release of calcium in skeletal muscle cells? A) Dantrolene sodium (Dantrium) B) Baclofen (Lioresal) C) Carisoprodol (Soma) D) Cyclobenzaprine (Flexeril)

A) Dantrolene sodium (Dantrium)

A patient is admitted to the emergency room in status epilepticus. What medication may be administered intravenously to assist in reducing seizure activity? A) Diazepam (Valium) B) Hydromorphone (Dilaudid) C) Insulin D) Meperidine (Demerol)

A) Diazepam (Valium)

A patient is taking tizanidine (Zanaflex) to treat spasticity from multiple sclerosis. Which of the following adverse effects of muscle relaxants is most pronounced with this medication? A) Hypotension B) Dark black urine C) Excessive salivation D) Eczema

A) Hypotension

A patient is admitted to the hospital with severe dehydration and also has decreased albumin levels. What effect will the patient's current status have if a prescribed dose of phenytoin (Dilantin) is administered? A) Potentially toxic serum level B) Reduced serum level C) Increased seizure activity D) Thromboembolism

A) Potentially toxic serum level

A 72-year-old male is newly diagnosed with Alzheimer's disease. The patient's daughter asks the nurse how her father's medication is going to help him. What would be the nurse's best response? A. "The drugs work by increasing ACh levels in the brain and slowing the progression of the disease." B. "The drugs work by crossing the blood-brain barrier and decreasing ACh levels in the brain." C. "The drugs work by decreasing ACh levels in the brain and slowing the progression of the disease." D. "The drugs work by crossing the blood-brain barrier and decreasing ACh levels in the neuromuscular junctions."

A. "The drugs work by increasing ACh levels in the brain and slowing the progression of the disease."

A patient is taking phenobarbital for a seizure disorder. Which of the following statements indicates that the patient should be seen by a health care provider immediately? A. "I have a rash that started on my trunk, and now it is on my arms and legs." B. "I rest if I feel tired." C. "I take my medication routinely and do not skip doses." D. "I have my blood levels checked if my breathing decreases."

A. A client taking phenobarbital who develops a rash may be developing Stevens-Johnson syndrome and should be seen by the primary health care provider immediately.

When describing the parasympathetic nervous system to a group of students, which substance would the nursing instructor discuss as being responsible for transmission of nerve impulses across this system? A. Acetylcholine B. Norepinephrine C. Dopamine D. Serotonin

A. Acetylcholine

A nurse should monitor a client taking donepezil (Aricept) for which adverse effects? (Select all that apply.) A. Anorexia B. Dizziness C. Headache D. Constipation E. Bradycardia

A. Anorexia B. Dizziness C. Headache

A client is brought to the Emergency Department having a cholinergic reaction, which includes a severe drop in blood pressure. What drug will the nurse expect to administer? A. Atropine (Generic) B. Edrophonium (Tensilon) C. Propranolol (Inderal) D. Succinylcholine (Anectine)

A. Atropine (Generic)

A nurse administers a peripherally acting muscle relaxant to a patient with a spinal cord injury. Which of the following drugs does the nurse administer? A. dantrolene B. baclofen C. methocarbamol D. diazepam

A. Dantrolene is a peripherally acting skeletal muscle relaxant.

The nuring instructor is teaching students about Alzheimer's disease. The instructor informs the students that clients with this illness experience problems with memory and thinking. The reason that this happens is which? A. Degeneration of cholinergic pathways B. Regenration of cholinergic pathways C. Increased level of acetylcholine D. Lack of neural destruction

A. Degeneration of cholinergic pathways

The client is ordered betanechol subcutaneously. After giving the client the injection of bethanechol, the nurse should make sure that what is available? A. Urinal, bed pan, and call light B. Magazine, book, or computer C. Sunglasses or eye mask D. Bowel or vomit bag

A. Urinal, bed pan, and call light

A nurse administers a peripherally acting muscle relaxant to a patient with a spinal cord injury. Which of the following drugs does the nurse administer? A. dantrolene (Dantrium) B. baclofen (Lioresal) C. methocarbamol (Robaxin) D. diazepam (Valium)

A. dantrolene (Dantrium) (all skeletal muscle relaxants except dantrolene are centrally acting drugs)

A cholinergic crisis is characterized by which event? A. excessive stimulation of the parasympathetic nervous system B. excessive stimulation of the sympathetic nervous system C. hypertension and respiratory failure D. stronger skeletal muscle contractions

A. excessive stimulation of the parasympathetic nervous system

When assessing a patient taking hydantoin therapy for seizure disorder, which indicates an adverse reaction to this therapy? A.Thrombocytopenia B.Leukocytosis C.Gingival atrophy D.Hypoglycemia

A.Thrombocytopenia (Severe side effects of hydantoins include thrombocytopenia, leukopenia, gingival hyperplasia, and hyperglycemia.)

Kate is experiencing status epilepticus. The prescriber orders IV diazepam. What special precautions need to be taken by a nurse to administer IV diazepam? Administer the drug very slowly, no faster than 5 mg/min. Administer the drug through an IV pump. Dilute the drug with 5 to 10 mL normal saline. Administer the drug in the small veins in the dorsum of the hand or the wrist.

Administer the drug very slowly, no faster than 5 mg/min. Explanation: Diazepam should be injected very slowly, no faster than 5 mg/min. Diazepam should not be mixed or diluted with other solutions or drugs, either in the syringe or in the intravenous bags of fluid. Since the small veins in the dorsum of the hand or the wrist should be avoided, diazepam can be administered through the IV infusion tubing as close to the insertion site into the vein as possible.

A client in status epilepticus has been brought to the emergency department. The nurse should anticipate which treatment measures? Administration of intravenous gabapentin Administration of intravenous lorazepam Administration of zonisamide (Zonegran) Administration of oral or sublingual phenytoin

Administration of intravenous lorazepam Explanation: Although diazepam has been the traditional treatment for status epilepticus, research has shown that lorazepam is more effective than diazepam or phenytoin used alone for cessation of seizures and has an effective duration of up to 6 hours compared to 20 minutes for diazepam. Oral medications are not administered during status epilepticus and neither zonisamide nor gabapentin are used to treat this health crisis.

A client in status epilepticus has been brought to the emergency department. The nurse should anticipate which treatment measures? Administration of intravenous lorazepam Administration of oral or sublingual phenytoin Administration of zonisamide (Zonegran) Administration of intravenous gabapentin

Administration of intravenous lorazepam Explanation: Although diazepam has been the traditional treatment for status epilepticus, research has shown that lorazepam is more effective than diazepam or phenytoin used alone for cessation of seizures and has an effective duration of up to 6 hours compared to 20 minutes for diazepam. Oral medications are not administered during status epilepticus and neither zonisamide nor gabapentin are used to treat this health crisis.

A client in status epilepticus has been brought to the emergency department. The nurse should anticipate which treatment measures?

Administration of intravenous lorazepam-Although diazepam has been the traditional treatment for status epilepticus, research has shown that lorazepam is more effective than diazepam or phenytoin used alone for cessation of seizures and has an effective duration of up to 6 hours compared to 20 minutes for diazepam. Oral medications are not administered during status epilepticus and neither zonisamide nor gabapentin are used to treat this health crisis.

A client has been admitted to the emergency department and is experiencing tonic-clonic seizures. What intervention should the nurse prioritize?

Administration of phenytoin IV as prescribed

A client has been admitted to the emergency department and is experiencing tonic-clonic seizures. What intervention should the nurse prioritize? Administration of phenytoin IV as prescribed Administration of gabapentin PO as prescribed Assessment of the client's renal and hepatic function Establishing a therapeutic relationship with the client

Administration of phenytoin IV as prescribed Explanation: Active seizure activity is an emergency and the client's immediate physiological needs supersede the importance of therapeutic relationship. Phenytoin IV is among the most common treatments for tonic-clonic seizures; gabapentin PO is not typically used for this purpose, and PO administration of any type is unsafe due to the risk for aspiration. Treatment would not be withheld pending assessment of renal and hepatic function.

An 18-year-old client has been taking phenytoin for the past 6 months for epileptic seizures. The client's phenytoin levels routinely fall within the therapeutic range. The client contacts the health care provider reporting nausea, headache, and diarrhea. What would be the most appropriate intervention based on the assessment of the client's symptoms and laboratory results? Advise the client that these are normal reactions to the medication and he should continue the medication as ordered. Advise the client to have his blood redrawn today to ensure that it is not at a toxic level. Advise the client to decrease the dose because the client is getting too much of the medication. Advise the client to double the dose for 24 hours because the client is experiencing the symptoms of drug withdrawal.

Advise the client to have his blood redrawn today to ensure that it is not at a toxic level. Explanation: Clients may demonstrate adverse effects even if the serum level of phenytoin is "normal"; it is important to adjust the dose to the clinical response of the client, not the serum level.

Which would a nurse identify as increasing a client's risk for hepatic disease with dantrolene use? Male gender Age over 35 years Respiratory disease Infection

Age over 35 years Explanation: The risk for hepatocellular disease is increased in women and all clients over the age of 35 years. Respiratory disease could be exacerbated with the use of dantrolene. Acute infection would be a contraindication to the use of botulinum toxins.

A nurse is caring for a patient who is going to take ethosuximide. During the nurse's initial assessment, the nurse learns that the patient's history includes hepatitis. Which laboratory test should be done before therapy is started? Blood urea nitrogen (BUN) Alanine aminotransferase (ALT) Partial thromboplastin time (PTT) Blood glucose level

Alanine aminotransferase (ALT) Explanation: Because an adverse effect of ethosuximide can be liver function impairment, a baseline liver function study that includes the ALT should be done prior to starting ethosuximide. Baseline values will help the nurse determine if the drug is causing the abnormal values, and liver function studies will need to continue throughout the therapy. Blood glucose level would determine the glucose level in the blood. BUN would help determine urinary function. PTT is a test to assess the intrinsic system, which is the common pathway of clot formation in the blood. While these tests are important, they do not have a direct relationship to ethosuximide use.

The nurse is caring for a client who is receiving an indirect-acting cholinergic medication and knows it is indicated as a treatment for myasthenia gravis as well as which other condition?

Alzheimer's disease Explanation: Indirect-acting cholinergic or anticholinesterase drugs are indicated to treat myasthenia gravis and Alzheimer's disease.

A patient with myasthenia gravis is administered neostigmine (Prostigmin). How does this drug produce its therapeutic effect? A) It stimulates the sympathetic nervous system. B) It stimulates the parasympathetic nervous system. C) It stimulates the cardiac smooth muscle. D) It stimulates the respiratory system.

Ans: B Feedback: Neostigmine (Prostigmin) stimulates the parasympathetic nervous system. It does not stimulate the sympathetic nervous system, cardiac system, or respiratory system.

Decreased acetylcholine levels are characteristic of what disorder?

Alzheimer's disease. Explanation: In normal brain function, acetylcholine is an essential neurotransmitter and plays an important role in cognitive functions, including memory storage and retrieval. Alzheimer's disease, the most common type of dementia in adults, is characterized by acetylcholine abnormalities. Multiple sclerosis (MS) is a disease of the central nervous system. Protecting the nerve fibers of the central nervous system is a fatty tissue called myelin, which helps nerve fibers conduct electrical impulses. In MS, myelin is lost in multiple areas, leaving scar tissue called sclerosis. Huntington's disease is a genetic neurologic disorder characterized by abnormal body movements called chorea and a lack of coordination; it also affects a number of mental abilities and some aspects of behavior. There is no specific test or marker for Parkinson's disease (PD); diagnosis is by a health care provider and depends on the presence of at least two of the three major signs: tremor at rest, rigidity, and bradykinesia, as well as the absence of a secondary cause, such as antipsychotic medications or multiple small strokes in the regions of the brain controlling movement.

A patient who is being treated for myasthenia gravis is receiving neostigmine, and her pulse drops to 50 after the administration. Which medication should be administered to treat the bradycardia? A) Atropine B) Pseudoephedrine C) Propranolol (Inderal) D) Bethanechol (Urecholine)

Ans: A Feedback: Atropine will reverse the muscarinic effects of cholinergic crisis. Pseudoephedrine will not affect bradycardia in this patient. Propranolol (Inderal) will not affect the bradycardia. Bethanechol will increase the bradycardia.

A patient is administered edrophonium (Tensilon) to confirm the diagnosis of myasthenia gravis. What effect will the edrophonium (Tensilon) have 30 minutes after the medication administration, which will confirm the diagnosis? A) Improved breathing B) Decreased fatigue C) Decreased muscle spasms D) Increased urinary output

Ans: A Feedback: Edrophonium (Tensilon) is used to diagnose myasthenia gravis. After administration, it will improve breathing in patients who have myasthenia gravis. Edrophonium will not decrease fatigue. Edrophonium will not decrease muscle spasms or increase urinary output.

A patient with myasthenia gravis is administered pyridostigmine (Mestinon) for the first time. She asks the nurse the reason why she has been changed to this medication. Which of the following is the nurse's best response? A) "Pyridostigmine (Mestinon) will allow you to awaken with the ability to swallow." B) "Pyridostigmine (Mestinon) can be taken every other day." C) "Pyridostigmine (Mestinon) is a faster-release form of cholinergic agent." D) "Pyridostigmine (Mestinon) is taken one time per day for better control."

Ans: A Feedback: Pyridostigmine is a slow-release form that is taken at bedtime and allows the patient the ability to swallow in the morning. Pyridostigmine is usually taken two times per day, not every other day. Pyridostigmine is a slow-release form, not a fast-release form. Pyridostigmine is taken two times per day, not one time per day.

A military nurse is part of a team reviewing the appropriate response to chemical weapons attacks, including sarin, tabun, and soman. When reviewing the physiology of a response to such drugs, the nurse should identify what goal of treatment? A) Restore anticholinesterase function. B) Restore GABA function. C) Restore dopamine function. D) Increase serotonin reuptake.

Ans: A Feedback: These drugs are irreversible anticholinesterase agents. The goal of treatment, such as atropine administration, is to restore normal function of anticholinesterase and rid synapses of excessive acetylcholine.

A patient has been receiving bethanechol (Urecholine) for 1 week. One hour after the dose is administered, he develops sweating, flushing, abdominal cramps, and nausea. What is the rationale for the development of these symptoms? A) Myasthenic crisis B) Cholinergic overdose C) Anaphylactic reaction D) Pulmonary edema

Ans: B Feedback: A patient who suffers cholinergic overdose will experience sweating, flushing, abdominal cramps, and nausea. An accurate diagnosis may be determined from timing in relation to medication. Signs and symptoms having an onset within approximately 1 hour after a dose of anticholinesterase drug are likely to be caused by cholinergic crisis. Signs and symptoms beginning 3 hours or more after a drug dose are more likely to be caused by myasthenic crisis. These symptoms are not the result of anaphylaxis or pulmonary edema.

A nurse is conducting health education with a man who has Alzheimer's disease and his daughter, who is his primary caregiver. The man has been deemed to be a good candidate for treatment with donepezil (Aricept) and will soon begin taking this medication. What teaching point should the nurse convey? A) "Aricept should be taken 1 hour before or 2 hours after any dairy products." B) "It's important to take this medication at bedtime." C) "Aricept should be taken with food to reduce the chance of stomach upset." D) "If you don't notice any cognitive improvement within 2 weeks, the drug should be discontinued."

Ans: B Feedback: Aricept should normally be taken at bedtime. It does not have to be taken with food. Dairy does not interfere with pharmacokinetics. The drug should be continued even if improvement is not noticeable.

A patient has atony of the smooth muscle of the gastrointestinal tract. Which type of medication may be administered to increase smooth muscle strength? A) Anticholinergic drugs B) Cholinergic drugs C) Muscle relaxants D) Selective serotonin reuptake inhibitors

Ans: B Feedback: Cholinergic drugs are used to treat atony of the smooth muscle of the gastrointestinal tract and urinary systems. Anticholinergic agents will decrease muscle strength. Muscle relaxants will decrease muscle strength. Selective serotonin reuptake inhibitors are not used for atony of the smooth muscle of the gastrointestinal tract.

A 33-year-old female patient with a recent history of visual disturbances and dysphagia has just been diagnosed with myasthenia gravis. The nurse should recognize that this patient's health problem is ultimately attributable to what pathophysiological process? A) Idiopathic overproduction of anticholinesterase B) Autoimmune destruction of acetylcholine receptors C) Demyelination of parasympathetic nerve pathways D) Deficient synthesis of dopamine by the substantia nigra

Ans: B Feedback: Myasthenia gravis occurs when antibodies produced by the body's own immune system block, alter, or destroy the receptors for acetylcholine at the neuromuscular junction, which prevents muscle contraction from occurring. The disease does not involve dopamine deficits, demyelination, or excess anticholinesterase.

A man with a neurogenic bladder secondary to a spinal tumor has been taking bethanechol. The patient has illuminated his call light, and the nurse has found him anxious, diaphoretic, and visibly flushed. Following a safety assessment, what action should the nurse take? A) Prepare to administer an IV dose of epinephrine to mitigate the effects of acetylcholine. B) Contact the physician as the patient may be experiencing a cholinergic crisis. C) Insert a Foley catheter as the patient may have excessive urine in his bladder. D) Assess the volume of the patient's bladder contents using a bladder ultrasound.

Ans: B Feedback: The presence of sweating and skin flushing in a patient taking bethanechol is suggestive of a cholinergic crisis. This is a medical emergency that warrants prompt intervention by the care team. This problem is not treated with epinephrine. Assessing the patient's bladder or inserting a Foley catheter is not sufficient.

A patient is diagnosed with a paralytic ileus. Which of the following medications will be administered to treat a paralytic ileus? A) Neostigmine (Prostigmin) B) Donepezil (Aricept) C) Bethanechol (Urecholine) D) Ambenonium (Mytelase)

Ans: C Feedback: Bethanechol (Urecholine) produces smooth muscle contractions and is used for obstructive conditions of the gastrointestinal tract. Neostigmine is used to treat myasthenia gravis. Donepezil is used to treat Alzheimer's disease. Ambenonium is used to treat myasthenia gravis.

A patient is diagnosed with Alzheimer's disease. The nurse should anticipate administering which of the following medications? A) Bethanechol (Urecholine) B) Neostigmine (Prostigmin) C) Donepezil (Aricept) D) Physostigmine salicylate (Antilirium)

Ans: C Feedback: Donepezil (Aricept) is used to treat mild to moderate Alzheimer's disease. Bethanechol (Urecholine) produces smooth muscle contractions and is not used to treat Alzheimer's disease. Neostigmine (Prostigmin) is not used to treat Alzheimer's disease; it is used to treat myasthenia gravis. Physostigmine is not used to treat Alzheimer's disease; it is an antidote for overdose of anticholinergic drugs.

A 40-year-old woman has received a diagnosis of myasthenia gravis (MG) and is scheduled to begin treatment with oral neostigmine in her home. When providing relevant health education, the nurse should emphasize that successful control of MG symptoms will primarily depend on A) matching each dose of neostigmine to the severity of symptoms at that time. B) making lifestyle modifications to prioritize a healthy diet and regular activity. C) adhering strictly to the prescribed administration schedule. D) developing acceptance of the functional deficits that result from MG.

Ans: C Feedback: Strict adherence to timely medication administration promotes optimal blood levels of neostigmine and optimal symptom control. Doses are not typically matched to shortterm symptom severity, except in emergencies. Lifestyle modifications and acceptance may be of benefit, but pharmacologic therapy is of paramount importance in the treatment of MG.

An 80-year-old man has been diagnosed with early-stage Alzheimer's disease and has begun treatment with donepezil (Aricept). When providing health education to the patient and his wife, the nurse should identify what goal of treatment? A) Remission of Alzheimer's disease B) Cure of Alzheimer's disease C) Improvement of cognition and function D) Resolution of memory and cognitive deficits

Ans: C Feedback: The goal of drug therapy for Alzheimer's disease is to slow the loss of memory and cognition, thus preserving the independence of the individual person for as long as possible. Remission, cure, and resumption of preillness levels of memory and cognition are unrealistic goals.

Following recent changes in memory and personality, a 72-year-old man is undergoing neurological testing to rule out Alzheimer's disease. The nurse is aware that this disease is characterized by what pathophysiological phenomena? Select all that apply. A) Meningiomas B) Neuritic plaques C) Neurofibrillary tangles D) Arteriovenous malformations (AVMs) E) Gliomas

Ans: C, D Feedback: The neuropathologic hallmarks of Alzheimer's disease are neuritic plaques and neurofibrillary tangles. Gliomas, meningiomas, and AVMs are not associated with Alzheimer's disease.

Which of the following medications is administered for ingestion of clitocybe mushrooms? A) Pyridostigmine (Mestinon) B) Donepezil (Aricept) C) Rivastigmine (Exelon) D) Atropine sulfate

Ans: D Feedback: Atropine sulfate is the specific antidote for mushroom poisoning. Pyridostigmine, donepezil, and rivastigmine are all cholinergic agents that would contribute to the symptoms of toxicity.

A middle-aged female patient has been admitted to the outpatient treatment unit of the hospital for an edrophonium (Tensilon) test. Shortly after the administration of the medication, the patient reports that her muscle strength is significantly weaker than before the test. The nurse who is participating in the test should recognize that this finding is suggestive of what diagnosis? A) Alzheimer's disease B) Anticholinergic crisis C) Myasthenia gravis D) Cholinergic crisis

Ans: D Feedback: If the edrophonium test makes the patient even weaker, the diagnosis is cholinergic crisis.

An older adult patient with a diagnosis of moderate Alzheimer's disease will soon return home with her husband. How can the nurse best facilitate the safe and effective administration of donepezil in the home setting? A) Have the patient explain the rationale for drug therapy repeatedly before discharge. B) Provide a detailed drug monograph to the patient and her husband and offer to answer any questions. C) Assess the patient's need for home care. D) Teach the patient's husband to administer the medication in a timely and safe manner.

Ans: D Feedback: It is important for the home care nurse to work with responsible family members in such cases to ensure accurate drug administration. Assessing the family's need for home care may be warranted, but this action does not necessarily ensure safe drug administration. The presence of moderate Alzheimer's disease may preclude effective patient teaching. Written teaching materials must normally be supplemented by other forms of teaching.

A client is receiving carbamazepine therapy and the client's latest serum level of the drug is 13 mcg/mL. What action is most appropriate? Anticipate a reduction in dosage Call an emergency code Contact the provider to request a one-time supplementary dose Assess the client's renal function

Anticipate a reduction in dosage Explanation: A serum carbamazepine level of 13 mcg/mL is slightly over the therapeutic range of 4 to 12 mcg/mL, indicating need for a reduction in dosage. This not an emergency, though it needs to be addressed. This result does not threaten the client's renal function.

A nurse would monitor a client closely for increased CNS depressant effects when a skeletal muscle relaxant is used concomitantly with which additional drug(s)? Select all that apply. Antihistamine Oral contraceptives Alcohol Opiates Antidiabetic medications

Antihistamine Alcohol Opiates The concomitant use of skeletal muscle relaxants and antihistamines, alcohol, opiates, or sedatives can result in increased CNS depressant effects. The use of a skeletal muscle relaxant and oral contraceptives may result in decreased effectiveness of the contraceptive. It is recommended that a barrier contraceptive be used to prevent transfer of the drug to the sperm and ensure effectiveness of the contraceptive. Skeletal muscle relaxants should be used cautiously by individuals using antidiabetic medication.

The 63-year-old client has been diagnosed with mild to moderate dementia. Which medications are prescribed for mild to moderate dementia? Select all that apply.

Aricept Razadyne Exelon Explanation: Aricept, Razadyne, and Exelon are all prescribed for mild dementia. Namenda is prescribed for moderate to severe dementia. Celexa is prescribed for depression, not dementia.

A new resident has just been admitted to the long-term care facility. When reviewing the client's medication record, the nurse observes that the client is currently taking donepezil. What should the nurse include in the resident's plan of care?

Assess the client's orientation each shift. Explanation: Donepezil is used to treat Alzheimer disease. Consequently, it would be necessary to monitor the client's cognition. Respiratory and musculoskeletal status would be less affected. Restraints are only used as an absolute last resort.

The nurse is working with a client who has been prescribed tizanidine for the treatment of muscle spasticity. When monitoring the client's risk for injury, what assessment should the nurse prioritize? assessment of bowel sounds and pattern of bowel movements assessment for headaches following peak blood levels Assessment of the client's creatinine clearance rate Assessment of the client's blood pressure following administration

Assessment of the client's blood pressure following administration Explanation: Tizanidine has been associated with hypotension, which could be a safety risk especially if the client is also taking an antihypertensive drug. Constipation and headaches are common adverse effects that do not pose as direct a safety risk. Creatinine levels are not normally affected, and these do not pose a safety risk.

The health care provider is preparing to administer edrophonium to confirm myasthenia gravis in a 47-year-old male client. What other drug must be available when edrophonium is administered?

Atropine Explanation: Edrophonium is a cholinergic agonist administered by a provider to diagnose myasthenia gravis. Use of this drug requires that an antidote, typically atropine, be on hand in case of cholinergic overdose.

A client experiences an overdose of a cholinergic drug. Which medication would the nurse anticipate that the client will receive as a reversal agent?

Atropine Explanation: Atropine is considered an antidote for a cholinergic drug overdose. Dopamine is a catecholamine neurotransmitter that is given to increase the contraction of the heart in shock. Epinephrine is a hormone released in the presence of stress and is given in the case of allergic reaction and norepinephrine is a neurotransmitter from the autonomic nervous system, that constricts blood vessels, raising blood pressure.

Which instruction should the nurse specifically stress when administering drugs used for muscle spasm and cramping? Avoid alcohol or other CNS depressants. Take the drug with food. Stay upright for 30 minutes after taking the drugs. Take the drug with 6 to 8 oz of water

Avoid alcohol or other CNS depressants. Explanation: The nurse should instruct the patient to avoid alcohol or other CNS depressants when taking a drug for muscle spasms and cramping. The nurse should instruct patients taking drugs for osteoporosis to take them with 6 to 8 oz of water and to stay upright for 30 minutes after taking drugs. The nurse should instruct patients with gout to take drugs for treating gout with food.

The nursing instructor is teaching a group of students about drugs used for muscle spasms and cramping. The instructor determines the session is successful when the students correctly choose which instruction as relevant to skeletal muscle relaxants?

Avoid alcohol or other CNS depressants.-The nurse should instruct the client to avoid alcohol or other CNS depressants when taking a drug for muscle spasms and cramping. The nurse should instruct clients taking drugs for osteoporosis to take them with 6-8 ounces of water and to stay upright for 30 minutes after taking drugs. The nurse should instruct clients with gout to take drugs for treating gout with food.

. A patient has a left temporal brain tumor. He smells an odor of ammonia prior to experiencing rapid rhythmic jerking movements. What is the odor of ammonia classified as? A) Chemical agent evoked by the tumor B) An aura prior to the seizure activity C) The metastatic process of tumor growth D) The inhibition of serotonin and acetylcholine

B) An aura prior to the seizure activity

A patient suffers from trigeminal neuralgia. What antiepileptic agent may be used to treat this disorder? A) Phenytoin (Dilantin) B) Carbamazepine (Tegretol) C) Fosphenytoin (Cerebyx) D) Ethosuximide (Zarontin)

B) Carbamazepine (Tegretol)

A patient is admitted with acute, painful muscle spasms and suffers from intermittent porphyria, an inherited enzyme deficiency. Which of the following muscle relaxants is contraindicated due to the patient's history of porphyria? A) Baclofen (Lioresal) B) Carisoprodol (Soma) C) Diazepam (Valium) D) Dantrolene (Dantrium)

B) Carisoprodol (Soma)

Which of the following medications is administered for status epilepticus? A. phenytoin B. diazepam C. levetiracetam D. clobazam

B. Diazepam is administered for status epilepticus.

The nurse is preparing a teaching plan for a client who is taking rivastigmine. What precaution should the nurse teach the client and family to minimize the risk of adverse GI effects? A. Monitoring weight throughout the therapy B. Having small, frequent meals C. Eating nutrient and calorie rich foods D. Establishing appropriate sleep and rest patterns

B. Having small, frequent meals

Which of the following places the patient at risk for toxicity following the administration of phenytoin? A. The patient skips a dose of phenytoin. B. The patient takes a different brand of phenytoin. C. The patient switches to a different antiepileptic agent. D. The patient receives phenytoin in an enteral feeding.

B. If a client switches brands of phenytoin, the client is at risk for the development of toxicity.

A patient is taking carisoprodol (Soma) for back spasms related to an occupational injury. Which of the following is most important to teach the patient? A. to take the medication at 8:00 am, 2:00 pm, and 8:00 pm B. to know the signs and symptoms of an idiosyncratic reaction C. to take the medication between meals D. to stop the medication with the first sign of abdominal cramping

B. to know the signs and symptoms of an idiosyncratic reaction

What is the highest priority nursing diagnosis for a patient taking phenytoin? A.Anxiety B.Risk for falls C.Risk for constipation D.Deficient fluid volume

B.Risk for falls (The nursing diagnosis "Risk for falls" has the highest priority for a patient taking phenytoin because it may lead to side effects of dizziness, decreased coordination, and ataxia. Anxiety, constipation, and efficient fluid volume are not side effects of phenytoin, but depression and discoloration of urine are.)

Before administering a daily dose of phenytoin, it is most important for the nurse to A.maintain the patient on bed rest. B.check phenytoin levels. C.monitor intake and output. D.monitor renal function tests.

B.check phenytoin levels. (Checking the phenytoin level is most important because of the narrow therapeutic range of 10 to 20 mcg/mL. Maintaining bed rest and monitoring I&O and renal function tests are not necessary.)

The nurse is caring for a client who is being discharged home from the rehabilitation unit. Baclofen will be discontinued, and the client will begin taking carisoprodol as an outpatient. What is the nurse's primary consideration when discontinuing the client's baclofen? Baclofen must be tapered down over 1 to 2 weeks to prevent psychoses and hallucinations. The client will be prescribed alternate doses of baclofen and carisoprodol over 10 days to prevent symptoms of baclofen withdrawal. The client's dose of baclofen should be gradually withdrawn over 4 to 6 weeks to prevent rebound spasticity. Carisoprodol will be administered while continuing baclofen until the carisoprodol level is established.

Baclofen must be tapered down over 1 to 2 weeks to prevent psychoses and hallucinations. Explanation: If using baclofen, taper drug slowly over 1 to 2 weeks to prevent the development of psychoses and hallucinations. Giving both drugs at once would risk toxicity and serious adverse effects and would never be done. Four to 6 weeks is unnecessary for the transition.

The nurse is caring for a client who is being discharged home from the rehabilitation unit. Baclofen will be discontinued, and the client will begin taking carisoprodol as an outpatient. What is the nurse's primary consideration when discontinuing the client's baclofen? Baclofen must be tapered down over 1 to 2 weeks to prevent psychoses and hallucinations. The client's dose of baclofen should be gradually withdrawn over 4 to 6 weeks to prevent rebound spasticity. Carisoprodol will be administered while continuing baclofen until the carisoprodol level is established. The client will be prescribed alternate doses of baclofen and carisoprodol over 10 days to prevent symptoms of baclofen withdrawal.

Baclofen must be tapered down over 1 to 2 weeks to prevent psychoses and hallucinations. Explanation: If using baclofen, taper drug slowly over 1 to 2 weeks to prevent the development of psychoses and hallucinations. Giving both drugs at once would risk toxicity and serious adverse effects and would never be done. Four to 6 weeks is unnecessary for the transition.

The nurse is caring for a client who is being discharged home from the rehabilitation unit. Baclofen will be discontinued, and the client will begin taking carisoprodol as an outpatient. What is the nurse's primary consideration when discontinuing the client's baclofen?

Baclofen must be tapered down over 1 to 2 weeks to prevent psychoses and hallucinations.- If using baclofen, taper drug slowly over 1 to 2 weeks to prevent the development of psychoses and hallucinations. Giving both drugs at once would risk toxicity and serious adverse effects and would never be done. Four to 6 weeks is unnecessary for the transition.

Which medication would the nurse expect to administer if prescribed to achieve skeletal muscle relaxation?

Baclofen-Baclofen is an example of a skeletal muscle relaxant.

The nurse is caring for an 84-year-old client in the acute care facility who was newly diagnosed with a seizure disorder. Before starting the client on an antiepileptic medication that will be continued after discharge, what laboratory studies should the nurse assess? Select all that apply.

Baseline kidney and liver function tests should be done and results will guide dosing of the antiepileptic medication because clients with liver or kidney disease will require lower dosages of medication. Serum drug levels will not be drawn until after specific medications are started. Cardiovascular and central nervous system function studies are not indicated.

A client with dementia of Alzheimer disease is prescribed memantine. Before administering the drug to the client, which would the nurse observe while obtaining the client's health history?

Behavior patterns Explanation: When taking the health history of the client, the nurse should observe the client's behavior patterns so that they can be used as a baseline for comparison during therapy. It is not required to observe eating habits, resistance power, and sleeping patterns when taking the health history of this client.

A client presents in the emergency department with tonic-clonic seizure activity. What is the IV drug of choice for treatment to obtain rapid control of the seizure? Topiramate Valproic acid Zonegran Benzodiazepine

Benzodiazepine Explanation: An IV benzodiazepine (e.g., lorazepam 0.1 mg/kg at 2 mg/min) is the drug of choice for rapid control of tonic-clonic seizures.

A nurse is reviewing the medication history of several clients. One of the clients is receiving a direct-acting cholinergic drug. The nurse would most likely identify which drug?

Bethanechol Explanation: Bethanechol is an example of a direct-acting cholinergic that acts like the neurotransmitter acetylcholine. Pyridostigmine, guanidine, and ambenonium are indirect-acting muscle stimulants.

The client has been diagnosed with urinary retention. The nurse expects the client to receive which medication for this condition?

Bethanechol Explanation: Bethanechol is the only medication listed that is used to treat urinary retention. Ambenonium and guanidine are used to treat myasthenia gravis. Benadryl is an antihistamine.

After teaching a group of nursing students about the various drugs used to treat musculoskeletal conditions, the instructor determines that the teaching was successful when the students correctly choose which drug as being used to treat osteoporosis? DMARDs Bone resorption inhibitors Skeletal muscle relaxants Uric acid inhibitors

Bone resorption inhibitors Explanation: Bone resorption inhibitors are used to treat osteoporosis. Disease-modifying antirheumatic drugs (DMARDs) are used to treat rheumatoid arthritis. Uric acid inhibitors are used to treat gout. Skeletal muscle relaxants are used to alleviate muscle spasms and cramping.

The nurse is caring for a client with asthmatic bronchitis and understands that cholinergic medications are contraindicated because this classification of medication may cause which effect?

Bronchoconstriction Explanation: Cholinergic drugs are contraindicated in people with asthma because they may cause bronchoconstriction and increased respiratory secretions.

A patient with muscle spasms is administered cyclobenzaprine (Flexeril). Which adverse effect should the nurse assess for with this medication? A) Muscle spasms B) Insomnia C) Drowsiness D) Urinary incontinence

C) Drowsiness

A patient has been taking phenytoin (Dilantin) for a seizure disorder. He has recently run out of his medication and has not obtained a refill. What is the patient at risk for developing? A) Hypotension B) Migraine headaches C) Status epilepticus D) Depression

C) Status epilepticus

The caregiver of client describes the client as having Alzheimer's disease (AD) for 10 years and is currently in the late stage of the disease. The caregiver asks the nurse if the client can go back on the medication donepezil. Which response by the nurse is appropriate? A. "Let me discuss your concerns with the prescriber." B. "A newer drug that has recently been developed is galantamine." C. "The drugs are best used early as they slow the progression of AD." D. "Donepezil has had research studies findings suggesting it cures AD."

C. "The drugs are best used early as they slow the progression of AD."

The nurse is caring for a client who is receiving an indirect-acting cholinergic medication and knows it is indicated as a treatment for myasthenia gravis as well as which other condition? A. Muscular dystrophy B. Musculoskeletal cancer C. Alzheimer's disease D. Cerebrovascular dementia

C. Alzheimer's disease

A nurse administers baclofen to a young man for a back injury. Which of the following interventions is most important related to the administration of baclofen? A. Assess heart rate. B. Assess for edema. C. Assess blood sugar. D. Assess for increased spasticity.

C. Baclofen has an adverse effect of hyperglycemia. It is most important to assess the client's blood sugar.

A client with dementia of Alzheimer disease is prescribed memantine. Before administering the drug to the client, which would the nurse observe while obtaining the client's health history? A. Eating habits B. Resistance power C. Behavior patterns D. Sleeping patterns

C. Behavior patterns

A client is experiencing urinary retention after surgery. The nurse would anticipate administering: A. Cevimeline B. Pilocarpine C. Bethanechol D. Carbachol

C. Bethanechol

A student asks the pharmacology instructor to describe the function of a cholinergic agonist. What would the instructor reply? A. Cholinergic agonists increase the activity of dopamine receptor sites throughout the brain and spinal cord. B. Cholinergic agonists decrease the activity of GABA receptor sites throughout the body. C. Cholinergic agonists increase the activity of acetylcholine receptor sites throughout the body. D. Cholinergic agonists decrease the activity of norepinephrine receptor sites throughout the brain and spinal cord.

C. Cholinergic agonists increase the activity of acetylcholine receptor sites throughout the body.

A patient is admitted to the emergency department with repeated tonic-clonic seizures. Tonic-clonic seizures may be attributed to which of following drugs? A. ciprofloxacin hydrochloride B. cimetidine C. cocaine D. morphine sulfate

C. Cocaine administration can result in seizure development.

Which of the following interventions assists the patient in decreasing the anticholinergic effects of cyclobenzaprine? A. Have the patient void before the administration of cyclobenzaprine. B. Give the patient lemon juice mixed with warm water to prevent constipation. C. Give the patient hard candy to suck on. D. Assess the patient's pulse and blood pressure.

C. Hard candy assists in relieving thirst related to anticholinergic effects of cyclobenzaprine.

Which of the following is most important to teach a patient who is being administered lamotrigine? A. to report the development of edema B. to report anxiety C. to report the development of a skin rash D. to report anorexia

C. It is necessary to instruct the client to report the development of skin rash when administered lamotrigine. The U.S. Food and Drug Administration has issued a black box warning concerning potential development of serious dermatologic reactions.

A nurse is conducting a medication resolution of a new resident of a care facility and notes that the woman has been taking neostigmine. The nurse should recognize that the woman may have a history of what health problem? A. Alzheimer disease B. Parkinson disease C. Myasthenia gravis D. Multiple sclerosis

C. Myasthenia gravis

A patient was discharged 3 days ago on phenytoin therapy for seizure disorder. The patient comes to the emergency department experiencing seizures. What will be of most value to determine the etiology of the returned seizures? A.A CT scan B.An EEG C.Serum phenytoin levels D.Serum electrolytes

C.Serum phenytoin levels (For dilantin therapy to be effective, a therapeutic serum range of 10 to 20 mcg/mL must be maintained. Subtherapeutic serum levels are a frequent cause of seizures for patients on dilantin therapy.)

A client's seizure activity, which is isolated to facial and neck muscles, has been identified as being caused by one area of the client's brain. The nurse should anticipate the administration of what drug?

Carbamazepine is often the drug of choice for treatment of partial seizures, which are described in this scenario. It has the ability to inhibit polysynaptic responses and to block sodium channels to prevent the formation of repetitive action potentials in the abnormal focus. Clorazepate is indicated for anxiety and alcohol withdrawal and used as adjunctive therapy for partial seizures. Felbamate has been associated with severe liver failure and aplastic anemia and is now reserved for those clients who do not respond to other therapies. Gabapentin is used as adjunctive therapy in the treatment of partial seizures and for the treatment of postherpetic neuralgia.

A client with a musculoskeletal disorder is unable to ambulate due to significant pain and immobility issues. Which care should the nurse prioritize for this client?

Changing the client's position every 2 hours

A toddler brought into the clinic by the parents is flaccid and cyanotic. The parents relate that just prior to arrival, the toddler was running a fever of 104.3°F (40.2°C) and began twitching uncontrollably. What is the nurse's first priority?

Check the airway.-Basic life support guidelines require evaluation of airway, breathing, and circulation as the nurse's first priority.

A nurse is caring for a 49-year-old client who has been taking bethanechol for treatment of neurogenic bladder. After one week, the client develops sweating, heavy salivation, and bronchospasm. What would the nurse suspect is happening with this client?

Cholinergic crisis Explanation: Overdose of cholinergic agonists can lead to cholinergic crisis. Symptoms of cholinergic crisis are those associated with excessive stimulation of the parasympathetic nervous system, including increased sweating, salivation, gastric secretions, and respiratory secretions. If not treated quickly, cholinergic crisis can lead to respiratory failure. It does not lead to myasthenic crisis, or pulmonary edema. With any medication there can be anaphylactic reaction, but these symptoms do not indicate that.

A nurse administering valproic acid (Depakote) to a client should monitor the client for increased effects and toxicity of valproic acid (Depakote) when which of the following medications are initiated? Select all that apply: Metformin (Glucophage) Amitriptyline (Elavil) Cimetidine (Tagamet) Lisinopril (Prinvil) Ibuprofen (Motrin)

Cimetidine (Tagamet) Amitriptyline (Elavil) Explanation: A nurse administering valproic acid (Depakote) to a client should monitor the client for increased effects and toxicity of valproic acid (Depakote) when antibiotics, antifungals, tricyclic antidepressants (amitriptyline), and cimetidine are initiated.

Which would be most important to monitor in a client receiving ethosuximide? Weight Nutritional status Electrocardiogram Complete blood count

Complete blood count Explanation: Although weight loss and anorexia may occur, ethosuximide is associated with bone marrow suppression, including potentially fatal pancytopenia, so it would be most important for the nurse to monitor the client's complete blood count. The drug is not associated with any cardiovascular effects that would necessitate an electrocardiogram.

A client is prescribed lamotrigine for control of partial seizures. What is the most important medication teaching information the nurse should emphasize from the plan of care?

Contact the health care provider immediately if rash appears.

The nurse is caring for a client who has just been prescribed cyclobenzaprine 100 mg PO t.i.d. What is the nurse's best action?

Contact the prescriber to confirm the dose.-The normal daily dosage of cyclobenzaprine is 10 mg taken orally t.i.d., and it can be increased to a maximum of 60 mg/day. As a result, the nurse must clarify this potentially unsafe dose.

A patient is being treated for intractable muscle spasticity with a peripherally acting spasmolytic. The nurse should anticipate administering which medication? Tizanidine Dantrolene Diazepam Cyclobenzaprine

Dantrolene Explanation: Dantrolene is a peripherally acting spasmolytic, while tizanidine, diazepam, and cyclobenzaprine are all centrally acting.

The client is taking cyclobenzaprine for muscle spasms secondary to an injury to the lumbar spine that occurred while lifting a motor at work. The client is being seen for a follow-up visit by the health care provider. The client reports dry mouth, blurred vision, and constipation. Why is the client having these side effects from cyclobenzaprine? Cyclobenzaprine has an effect at the neuromuscular junction Cyclobenzaprine acts in the peripheral nervous system Cyclobenzaprine produces an anticholinergic response Cyclobenzaprine is structurally similar to amitriptyline

Cyclobenzaprine produces an anticholinergic response Explanation: Cyclobenzaprine relieves muscle spasms through a central action, possibly at the level of the brain stem, with no direct action on the neuromuscular junction or the muscle involved. The common adverse effects of cyclobenzaprine are related to its CNS depression and anticholinergic activity. The most common adverse effects are drowsiness, dizziness, and dry mouth.

A patient who has been taking valproic acid (sodium valproate) for a seizure disorder is asking the nurse about getting pregnant. Why is pregnancy discouraged in women who are being treated for seizure disorders? A) Seizure disorders are genetic. B) Seizure disorders are familial. C) Antiepilepsy drugs decrease fertility. D) Antiepilepsy drugs are teratogenic.

D) Antiepilepsy drugs are teratogenic.

. A patient has been diagnosed with a brain tumor, which has caused partial seizure activity. The patient is being treated with gabapentin (Neurontin). After administering the medication, the nurse should assess the patient because of the potential for what adverse effect? A) Tetany B) Hypersensitivity C) Paradoxical seizures D) CNS depression

D) CNS depression

A patient is admitted to the hospital for severe back spasms and pain. Which of the following skeletal muscle relaxants will act peripherally on the muscle itself? A) Cyclobenzaprine (Flexeril) B) Carisoprodol (Soma) C) Methocarbamol (Robaxin) D) Dantrolene sodium (Dantrium)

D) Dantrolene sodium (Dantrium)

An 80-year-old patient has severe pain after a case of shingles. The pain is noted along the shoulder and back. He states the pain is so severe he cannot sleep. What is the primary medication that will relieve this pain? A) Meperidine (Demerol) B) Morphine sulfate (MS Contin) C) Naproxen sodium (Naprosyn) D) Gabapentin (Neurontin)

D) Gabapentin (Neurontin)

A patient has been started on dantrolene (Dantrium). What is the most serious adverse effect about which the patient should be instructed? A) Metabolic acidosis B) Hypercarbia C) Renal calculi D) Hepatitis

D) Hepatitis

A child suffers from absence seizures and has been prescribed acetazolamide (Diamox). The nurse should know that this medication is an adjuvant medication useful in the treatment of seizures. What is the therapeutic action of Diamox? A) It slows the action potential of neurons. B) It slows the reuptake of acetylcholine. C) It suppresses the limbic and reticular systems. D) It controls fluid secretion in the CNS.

D) It controls fluid secretion in the CNS.

A patient with impaired liver function is suffering from a seizure disorder that most often results in partial seizures. Which of the following AEDs may be administered to a patient with impaired liver function? A) Oxcarbazepine (Trileptal) B) Fosphenytoin (Cerebyx) C) Carbamazepine (Tegretol) D) Levetiracetam (Keppra)

D) Levetiracetam (Keppra)

A patient is having a seizure that lasts longer than 30 minutes. What type of generalized seizure is the patient experiencing? A. tonic-clonic B. absence seizure C. atonic seizure D. status epilepticus

D. A client having a seizure lasting longer than 30 minutes is experiencing status epilepticus.

What drug would be prescribed for treatment of abdominal distension related to a paralytic ileus? A. neostigmine B. pyridostigmine C. edrophonium D. bethanechol

D. bethanechol

The nurse is caring for a 15-year-old male who was involved in a motor vehicle accident and, as a result, sustained a closed head injury. The health care provider ordered phenytoin prophylactically to prevent seizures. The nurse understands that phenytoin works by what mechanism? Decreases the sodium influx into the cell, thereby preventing the cell from producing a stimulus Increases the chloride available to promote depolarization of the cells Increases the potassium available to the cell to reduce the repolarization of the cell Decreases the calcium available to the cells responsible for electrical activity in the brain

Decreases the sodium influx into the cell, thereby preventing the cell from producing a stimulus Explanation: Phenytoin (Dilantin) is the prototype drug that controls seizures by decreasing sodium influx into the cells. Sodium influx produces an action potential, which then causes the neurons to fire.

The nurse is administering phenytoin to a client who is also receiving a continuous nasogastric enteral feeding and is aware of what possible effect? Increasing absorption of the AED Decreasing the absorption of the AED Not affecting absorption of the AED Precipitating signs of overdosage

Decreasing the absorption of the AED Explanation: The use of continuous nasogastric enteral feedings may decrease the absorption of phenytoin administered through the same route, predisposing the client to the risk of seizure activity.

The nurse is administering phenytoin to a client who is also receiving a continuous nasogastric enteral feeding and is aware of what possible effect?

Decreasing the absorption of the AED-The use of continuous nasogastric enteral feedings may decrease the absorption of phenytoin administered through the same route, predisposing the client to the risk of seizure activity.

A client with Alzheimer's disease asks the nurse what the common adverse reactions to cholinesterase inhibitors are. What common side effects could the nurse provide? Select all that apply.

Dizziness Headache Diarrhea Explanation: Common adverse reactions to cholinesterase inhibitors include: diarrhea, headache, and dizziness. Seizure and flatulence are not common adverse reactions.

What is the most important teaching from the plan of care, to emphasize, when a client is prescribed a skeletal muscle relaxant? Report increased muscle spasm to the health care provider immediately. Always take this medication with food to prevent nausea and vomiting. Do not drive or operate machinery until individual drug effects are known. Drink eight to 10 glasses of water daily to prevent constipation.

Do not drive or operate machinery until individual drug effects are known. Explanation: Drowsiness is the most common reaction seen with the use of skeletal muscle relaxants. The priority of care is safety; therefore, the most important teaching is to tell the client not to drive or operate machinery until the client knows how the drug affects him/her individually. Nausea and constipation may occur, but are not safety priorities. Increased muscle spasm may indicate the drug is not effective, but is not a high safety concern.

The nurse is reviewing the results of a hospital client's serum phenytoin level, which has just become available. The results indicate that the client's phenytoin level is 17.5 mcg/mL. What is the nurse's best action? Document the fact that the nurse checked the client's phenytoin levels Raise the client's bed rails and maintain the client on bed rest Contact the care provider to communicate this result Perform a focused neurological assessment

Document the fact that the nurse checked the client's phenytoin levels Explanation: The therapeutic range is 10 to 20 mcg/mL. Consequently, there is no action needed beyond documentation.

Which medication would the nurse expect to administer orally once a day to a client with AD?

Donepezil Explanation: Donepezil is given once a day. While rivastigmine, galantamine, and memantine are given twice a day.

A nurse administers carisoprodol to a client for the treatment of an acute musculoskeletal condition. The nurse would be alert for which adverse effect after administering the drug? Drowsiness Anxiety Constipation Insomnia

Drowsiness Explanation: The nurse should monitor for drowsiness in the client as the adverse reaction of carisoprodol administration. Depression is not an adverse reaction of carisoprodol administration. Insomnia and anxiety are adverse reactions associated with bisphosphonate drugs.

When developing a teaching plan for a client who is to receive carisoprodol, which sign or symptom would the nurse include as a common adverse reaction? Drowsiness Dyspnea Hypertension Tachycardia

Drowsiness Explanation: Drowsiness is the most common adverse reaction to skeletal muscle relaxants like carisoprodol that the nurse should discuss with the client. No correlation is found with skeletal muscle relaxants causing dyspnea. The Disease-modifying antirheumatic medication of leflunomide has the adverse reaction of hypertension. Tachycardia can be seen in the use of skeletal muscle relaxants but is not the most common and is seen in the use of dantrolene and diazepam.

When reviewing a journal article about seizure disorders, the nurse would expect to find tonic-clonic seizures and myoclonic seizures being classified as which type of seizures? Complex seizures Atonic seizures Generalized seizures Partial seizures

Generalized seizures Explanation: Tonic-clonic seizures and myoclonic seizures are classified as generalized seizures. Partial seizures can be simple or complex. Complex seizures are partial seizures that involve impaired consciousness and variable unconscious repetitive actions, staring gaze, and hallucinations/delusions. Atonic seizures are a type of generalized seizure involving the loss of muscle tone where the person suddenly drops.

Which type of seizures involves a loss of consciousness?

Generalized seizures-

When reviewing a journal article about seizure disorders, the nurse would expect to find tonic-clonic seizures and myoclonic seizures being classified as which type of seizures?

Generalized seizures- Tonic-clonic seizures and myoclonic seizures are classified as generalized seizures. Partial seizures can be simple or complex. Complex seizures are partial seizures that involve impaired consciousness and variable unconscious repetitive actions, staring gaze, and hallucinations/delusions. Atonic seizures are a type of generalized seizure involving the loss of muscle tone where the person suddenly drops.

An older adult client has been prescribed an antiseizure medication and is experiencing central nervous system depression. What is the nurse's most appropriate action? Make a referral to occupational therapy Implement falls precautions Maintain the client on bed rest Monitor vital signs hourly while the client is awake

Implement falls precautions Explanation: CNS depression creates a risk for falls, especially in older clients. There is no need to assess vital signs on an hourly basis and the harm of bed rest exceeds the benefits. Occupational therapy has no direct relationship with treating CNS depression.

An older adult client has been prescribed an antiseizure medication and is experiencing central nervous system depression. What is the nurse's most appropriate action?

Implement falls precautions-CNS depression creates a risk for falls, especially in older clients.

The nurse is caring for a client whose current medication regimen includes baclofen 60 mg PO daily. What assessment should the nurse prioritize when assessing for therapeutic effects? Assessment for seizure activity Inspection for muscle spasticity and range of motion assessment Comparison of fine motor and gross motor skills Bilateral assessment of muscle strength

Inspection for muscle spasticity and range of motion assessment Explanation: Baclofen is prescribed to treat muscle spasticity and/or acute musculoskeletal discomfort. It is not an anticonvulsant and does not affect fine motor versus gross motor skills. An absence of spasticity may increase muscle strength but this is not the most direct effect of the medication.

A client has received a prescription for baclofen. The home care nurse would schedule which laboratory tests to monitor this client? CBC and electrolytes Liver function tests Hemoglobin and hematocrit Cardiac function tests

Liver function tests Explanation: Baclofen is metabolized in the liver and excreted in urine. The client must be monitored for adverse effects on liver function.

Which would a nurse include when describing the action of dantrolene? Interference with calcium release from the muscles Inhibition of the release of acetylcholine Interference with the reflexes causing the spasm Inhibition of presynaptic motor neurons in central nervous system

Interference with calcium release from the muscles Explanation: Dantrolene interferes with the release of calcium from the muscle tubules, preventing the fibers from contracting. Botulinum toxins A and B bind directly to the receptor sites of motor nerve terminals and inhibit the release of acetylcholine. Centrally acting skeletal muscle relaxants interfere with the reflexes that are causing the muscle spasm. Tizanidine is thought to increase inhibition of presynaptic motor neurons in the CNS.

A client with spastic hypertonia resulting from cerebral palsy has not responded appreciably to oral baclofen. During the client's care conference, the nurse should anticipate a discussion about what possible treatment? Intrathecal baclofen infusion Cyclobenzaprine extended release Phenytoin IV infusion Phenobarbital IV intermittently

Intrathecal baclofen infusion Explanation: Baclofen is available by intrathecal administration for intractable spasticity. Oral cyclobenzaprine would not likely be used is a client has not responded to other oral centrally acting skeletal muscle relaxants. Phenytoin and phenobarbital treat seizure activity, not muscle spasticity.

A primary health care provider has prescribed phenytoin parenterally for a client experiencing seizures. The nurse prepares to administer this drug via which route? Intradermally Subcutaneously Intramuscularly Intravenously

Intravenously Explanation: Phenytoin can be administered orally and parenterally. If the drug is administered parenterally, the intravenous (IV) route is preferred over the intramuscular (IM) route because, with the IM route, erratic absorption of phenytoin causes pain and muscle damage at the injection site. The drug is not administered intradermally or subcutaneously.

A client is receiving allopurinol. Which instruction would be most important for the nurse to include in the client's plan of care? Liberal fluid intake Moderate exercise Use of a brace or corset Avoidance of direct sunlight

Liberal fluid intake Explanation: When using uric acid inhibitors, such as allopurinol, the nurse should encourage liberal fluid intake and measure the client's intake and output. The client does not need to exercise or use braces or corsets; clients with osteoporosis may require a brace or corset when out of bed. The client need not avoid sunlight as uric acid inhibitors do not cause photosensitivity.

What should the nurse include as a possible adverse effect when teaching a client about phenytoin?

Liver toxicity- Liver toxicity is a potential adverse effect of phenytoin. Constipation, not diarrhea, is an adverse effect of phenytoin. Bone marrow suppression and leukopenia would be adverse effects of phenytoin. Physical dependence is an adverse effect associated with the use of benzodiazepines and barbiturates.

An operating room nurse is assisting the anesthesiologist in the preparation of an intravenous dose of dantrolene. What emergent issue most likely prompted the need to give the patient dantrolene? Malignant hyperthermia Uncontrolled hypertension Myocardial ischemia Cerebrovascular accid

Malignant hyperthermia Explanation: IV dantrolene is the drug of choice, when accompanied by supportive measures, for acute treatment of malignant hyperthermia. The drug is not used in the treatment of emergent CVA, myocardial ischemia, or hypertension

A nurse is caring for a patient with seizure disorders. The patient has been prescribed phenytoin. Which of the following ongoing assessment activities should a nurse perform during the treatment? Check for the patient's vital signs every three hours. Measure regular serum plasma levels of the drug. Measure the respiratory rate of the patient. Measure the pulse rate of the patient.

Measure regular serum plasma levels of the drug. Explanation: While caring for the patient on anticonvulsant treatment, the nurse should measure the regular serum plasma levels of the drug. The nurse need not check for the patient's vital signs every three hours, measure the respiratory rate of the patient, nor measure the pulse rate of the patient while caring for a patient on anticonvulsant treatment.

The nurse is caring for a client with Alzheimer's disease who has been prescribed a cholinesterase inhibitor. What symptom of the disorder does the nurse expect the medication is intended to treat?

Mild to moderate dementia Explanation: Cholinesterase inhibitors are used to treat mild to moderate dementia, not other symptoms of AD such as wandering, irritability, or aggression. The anticipated therapeutic effects are improved memory and reduction of dementia.

When describing the action of direct-acting cholinergic agonists, which receptors would the nurse identify as being stimulated?

Muscarinic Explanation: Direct-acting cholinergic agonists usually stimulate muscarinic receptors within the parasympathetic nervous system. Alpha and beta receptors are found in the sympathetic nervous system and are affected by adrenergic agents.

A nurse should alert a client to which adverse effects of pyridostigmine ? (Select all that apply.)

Nausea Skin flushing Explanation: Pyridostigmine is an oral cholinergic medication used in the treatment of myasthenia gravis. General adverse reactions associated with oral administration include nausea, diarrhea, abdominal cramping, salivation, skin flushing, cardiac arrhythmias, and muscle weakness

A 40-year-old female client with myasthenia gravis has been prescribed dantrolene. The nurse has taught the client about the risks of liver toxicity associated with the medication and should teach the client to self-monitor for what early indications? Nausea and loss of appetite Weight gain and ankle edema Nosebleeds or bleeding gums Flank pain and abdominal distention

Nausea and loss of appetite Explanation: The prodrome associated with liver toxicity during dantrolene therapy includes nausea, anorexia and fatigue. It is not characterized by flank pain, weight gain, abdominal distention, edema or bleeding.

Imbalanced Nutrition: Less Than Body Requirements is a common nursing diagnosis for clients prescribed cholinesterase inhibitors. The client's daughter asks the nurse why this is common. The nurse knows that which is a common side effect of cholinesterase inhibitors?

Nausea and vomiting Explanation: Nausea and vomiting are common side effects that cause weight loss in a client taking cholinesterase inhibitors. Hypotension, hypertension, and hyperactivity are not common side effects.

After teaching a group of students about indirect acting cholinergic agonists used to treat myasthenia gravis, the instructor determines that the teaching was effective when the students identify which agent?

Neostigmine Explanation: Neostigmine is used to treat myasthenia gravis. Galantamine is used to treat Alzheimer's disease. Rivastigmine is used to treat Alzheimer's disease. Tacrine is used to treat myasthenia gravis

A client asks the nurse about the cause of idiopathic seizures. What is the nurse's best response? Passed from parent to child No known cause Brain injury at birth High fever

No known cause Explanation: An idiopathic seizure has no known cause. Hereditary seizure disorders are passed from parent to child in their genetic makeup. Acquired seizure disorders have a known cause such as high fever. Brain injury at birth is an example of a cause for epilepsy.

A nurse is caring for a patient prescribed phenobarbital for status epilepticus. What intervention should the nurse perform when the patient has been administered the drug? Observe respirations frequently. Monitor blood glucose levels. Record fluid input and output. Monitor body temperature.

Observe respirations frequently. Explanation: When caring for a patient who has been administered phenobarbital, the nurse should observe respirations frequently. The nurse need not monitor blood glucose levels or body temperature. The nurse need not record fluid input and output. The nurse may need to observe blood glucose levels if the patient is being administered antidiabetic medications along with an anticonvulsant.

A nurse is caring for a patient prescribed phenobarbital for status epilepticus. What intervention should the nurse perform when the patient has been administered the drug?

Observe respirations frequently.-When caring for a patient who has been administered phenobarbital, the nurse should observe respirations frequently.The nurse may need to observe blood glucose levels if the patient is being administered antidiabetic medications along with an anticonvulsant.

A client is to receive ethotoin. The nurse would expect to administer this drug by which route?

Oral

A client is to receive ethotoin. The nurse would expect to administer this drug by which route? Rectal Intramuscular Oral Intravenous

Oral Explanation: Ethotoin is administered orally.

The nurse has administered anakinra to a client. Which assessment finding should the nurse prioritize? Constipation Abdominal pain Retinal changes Pancytopenia

Pancytopenia Explanation: Due to the immunosuppressive properties of DMARD drugs, pancytopenia is an adverse effect of anakinra, a DMARD, and the client should be monitored for it closely. Administration of anakinra may also cause headache and irritation at the injection site, but not constipation, abdominal pain, or retinal changes. Constipation is a potential adverse reaction to cyclobenzaprine. Retinal changes are potential adverse reactions to hydroxychloroquine. Pancytopenia is a potential adverse reaction to sulfasalazine, alendronate, ibandronate, risedronate, allopurinol, and colchicine.

The nurse has administered anakinra to a client. Which assessment finding should the nurse prioritize?

Pancytopenia-Due to the immunosuppressive properties of DMARD drugs, pancytopenia is an adverse effect of anakinra, a DMARD, and the client should be monitored for it closely. Administration of anakinra may also cause headache and irritation at the injection site, but not constipation, abdominal pain, or retinal changes. Constipation is a potential adverse reaction to cyclobenzaprine. Retinal changes are potential adverse reactions to hydroxychloroquine. Pancytopenia is a potential adverse reaction to sulfasalazine, alendronate, ibandronate, risedronate, allopurinol, and colchicine.

The nursing instructor is discussing cholinergic drugs with his clinical group. What system would the instructor tell the students is stimulated by cholinergic drugs?

Parasympathetic nervous system Explanation: Cholinergic agonists act at the same site as the neurotransmitter acetylcholine (ACh) and increase the activity of the ACh receptor sites throughout the body. Because these sites are found extensively throughout the parasympathetic nervous system, their stimulation produces a response similar to what is seen when the parasympathetic system is activated.

A nursing student is preparing to administer the drug bethanechol to a client with urinary retention. After researching the drug's actions and uses, the student demonstrates understanding of the drug by identifying it as which type of medication? Select all that apply.

Parasympathomimetic drug Cholinergic drug Explanation: Bethanechol is an example of a parasympathomimetic drug because it mimics the activity of the parasympathetic nervous system. Bethanechol is a cholinergic drug that mimics the activity of the parasympathetic nervous system. An adrenergic blocker interferes with the action of the sympathetic nervous system. A sympatholytic drug blocks or inhibits the action of the sympathetic nervous system.

A male client routinely takes baclofen as a skeletal muscle relaxant for a neuromuscular disorder. His last lab results indicate that he is experiencing renal insufficiency. Based on these data, what would the nurse expect the health care provider to do? Increase the dose Titer the dose Maintain the current dose Reduce the dose

Reduce the dose Explanation: The skeletal muscle relaxants should be used cautiously in clients with renal impairment. Dosage of baclofen must be reduced.

After teaching a group of nursing students about seizures, the instructor determines that the teaching was successful when the group identifies seizures that do not impair consciousness but can involve the senses or motor ability as which type? Partial seizures Generalized seizures Tonic-clonic seizures Myoclonic seizures

Partial seizures Explanation: Seizures that do not impair consciousness but can involve the senses or motor ability are classified as partial seizures. Generalized seizures involve loss of consciousness during the seizure. Tonic-clonic seizures are a type of generalized seizure involving alternate contraction and relaxation of the muscles, loss of consciousness, and abnormal behavior. Myoclonic seizures involve sudden, forceful contractions of single or multiple groups of muscles.

A nurse is caring for a patient with urinary retention. The patient has been prescribed Urecholine. For which category of patients should the nurse take precautions when administrating Urecholine?

Patients with bradycardia Explanation: The nurse should administer Urecholine cautiously in patients with bradycardia, hypertension, epilepsy, cardiac arrhythmias, recent coronary occlusion, and megacolon. The nurse need not take precautions for the patients with Raynaud's disease, recent myocardial infarction, and hyperthyroidism when administrating Urecholine.

A nurse is administering lorazepam to a client with status epilepticus. The nurse will be prepared to administer which additional drug to treat the status epilepticus for the next several hours?

Phenytoin

The client, newly diagnosed with epilepsy, begins to show signs of seizure activity. What is the nurse's priority action? Obtain the client's vital signs as soon as possible. Attempt to calm the client. Protect the client's safety. Perform detailed documentation.

Protect the client's safety. Explanation: As in any nursing situation, safety is paramount. The client's safety is prioritized over documentation or gathering vital signs. Seizure activity cannot be influenced by calming a client.

The nurse has completed a teaching with a client prescribed phenytoin daily for seizure activity. The nurse determines the session is successful after the client correctly chooses which potential adverse reaction that can occur if the client misses a dose of medication?

Recurrence of seizures-Recurrence of seizure activity may result from abrupt discontinuation of the drug, even when the anticonvulsant is being administered in small daily doses. Abrupt discontinuation of the drug does not cause CNS depression, hypotension, or nystagmus. CNS depression, hypotension, and nystagmus are adverse reactions of phenytoin.

A nurse is preparing to administer carbamazepine to a client. The nurse should question the health care provider about this order if which disorder is noted in the client's history?

Renal impairment

While taking the vital signs of a hospitalized client admitted for seizure control due to epilepsy, the nurse notices a bloody toothbrush on the client's bedside table and scattered bruising over the client's extremities. What is the nurse's best action? Tell the client to ask for assistance when ambulating so as to prevent bruising. Report the findings to the primary health care provider immediately. Document the findings and offer the client a soft-bristled toothbrush . Notify the charge nurse about reporting suspected physical abuse.

Report the findings to the primary health care provider immediately. Explanation: The client has a history of epilepsy and is likely taking anticonvulsants. Hematologic changes when taking anticonvulsants needs to be reported immediately to the health care provider. Such changes include bleeding gums and easy bruising. Abuse is not the likely cause of the client's bruising given the history of epilepsy. Telling the client to ask for assistance when ambulating and offering a soft toothbrush are appropriate, but the health care provider needs to be notified immediately about the hematologic changes.

The nurse is teaching a client about the medication they are receiving as treatment for gout. The nurse determines the session is successful when the client correctly chooses which instruction as most important? Taking drug on an empty stomach Using protection against sunlight Reporting any skin rash Wearing a brace to get out of bed

Reporting any skin rash Explanation: The nurse should instruct the client to report any skin rash. A rash should be monitored carefully because it may precede a serious adverse reaction, such as Stevens-Johnson syndrome. The nurse need not instruct the client to take the drug on an empty stomach, use protection against sunlight, or wear a brace to get out of bed. Clients with osteoporosis are asked to wear a brace to get out of bed. Clients taking medications for gout are asked to take it with food. These clients are also instructed to avoid driving or performing other hazardous tasks.

A nurse is developing a plan of care for a client receiving anticonvulsant therapy and determines a nursing diagnosis of risk for injury is most appropriate. Which assessment finding(s) would support this nursing diagnosis? Select all that apply. Epistaxis Reports of blurred vision Reports of dizziness Photosensitivity Scaling red rash

Reports of blurred vision Reports of dizziness Photosensitivity

An older adult client experiencing seizures is prescribed diazepam. Which assessment should the nurse prioritize? Respiratory rate and depth Blood glucose levels Swallowing ability Speech quality

Respiratory rate and depth Explanation: Apnea and cardiac arrest can occur when diazepam is administered to older adults, very ill clients, and individuals with limited pulmonary reserve. Therefore, monitoring the client's respiratory rate and depth would be most important. If the client is prescribed diazepam and antidiabetic medications, there may be an increase in the blood glucose level. The nurse should test the client's swallowing ability by offering sips of water before giving the drug. If there is difficulty swallowing, hold the drug and contact the health care provider as soon as possible. Slurred speech is an adverse reaction of hydantoins. It is also a potential sign of toxicity; however, the most important would be to determine the respiratory status and if that is functioning within reasonable parameters, then address the toxicity issue, if present.

A client has been prescribed phenytoin for the treatment of seizures. How should the nurse most accurately determine whether the client has therapeutic levels of the medication?

Review the client's laboratory blood work.

A client has been prescribed phenytoin for the treatment of seizures. How should the nurse most accurately determine whether the client has therapeutic levels of the medication? Review the client's laboratory blood work. Assess the client carefully for adverse effects. Monitor the client for seizure activity. Assess the client's cognitive status.

Review the client's laboratory blood work. Explanation: Measuring serum drug levels evaluates whether the therapeutic range of circulating drug can be found in the serum. It does not directly evaluate effectiveness of therapy, however, which can only be evaluated by determining whether the drug is having the desired effect of reducing number of seizures. Short-term absence of seizures does not necessarily indicate that drug is within therapeutic range.

The nurse is developing a teaching plan for an outpatient receiving cholinergic drugs. What should be the role of the nurse when developing a teaching plan?

Review the purpose of the drug therapy with the patient and family. Explanation: The nurse should review the purpose of the drug therapy with the patient and family when developing a teaching plan for an outpatient receiving cholinergic drugs. Suggesting that the patient avoid fiber-rich food during therapy, suggesting the patient adapt a self blood pressure measurement habit, and reviewing the patient's previous history of disorders are not related roles of the nurse when developing a teaching plan for an outpatient receiving cholinergic drugs.

When reviewing a client's history, which condition would the nurse identify as contraindicating the use of a centrally acting skeletal muscle relaxant? Rheumatic disorder Hypertension Epilepsy Depression

Rheumatic disorder Explanation: Centrally acting skeletal muscle relaxants would be contraindicated for treatment of muscle spasms related to a rheumatic disorder. Epilepsy, depression, or hypertension would not necessitate cautious use.

A 77-year-old client is being admitted to a long-term care facility. The client has a history of absence seizures has been treated with ethosuximide for many years. While the nurse is creating a plan of care on the client, the nurse understands the potential adverse effects of this drug and would consequently prioritize which nursing diagnoses? Risk for Impaired Skin Integrity Risk for Impaired Gas Exchange Risk for Altered Urinary Elimination Risk for Falls

Risk for Falls Explanation: Common adverse effects of ethosuximide are drowsiness, dizziness, and lethargy- which potentiates the risk for fall in this client. Respiration, urinary function, and skin integrity are not normally affected by the use of ethosuximide.

A 77-year-old client is being admitted to a long-term care facility. The client has a history of absence seizures has been treated with ethosuximide for many years. While the nurse is creating a plan of care on the client, the nurse understands the potential adverse effects of this drug and would consequently prioritize which nursing diagnoses?

Risk for Falls- Common adverse effects of ethosuximide are drowsiness, dizziness, and lethargy- which potentiates the risk for fall in this client. Respiration, urinary function, and skin integrity are not normally affected by the use of ethosuximide.

A 40-year-old client with a diagnosis of fibromyalgia has been prescribed cyclobenzaprine (Flexeril) as an adjunct to her existing drug regimen. What nursing diagnosis should the nurse prioritize for the nursing care plan for this client? Diarrhea related to anticholinergic effects Altered Nutrition, Less than Body Requirements, related to appetite suppression Impaired Swallowing related to increased muscle tone Risk for Injury related to CNS depressant effects

Risk for Injury related to CNS depressant effects Explanation: The CNS depression that is associated with the use of cyclobenzaprine constitutes a risk for injury. The anticholinergic effects of the drug constitute a risk for constipation, not diarrhea. Nutrition and swallowing are not typically affected by the use of cyclobenzaprine.

A client with gout is prescribed pegloticase after not responding to the usual medications. Which nursing diagnosis would the nurse determine to be most appropriate in this case? Risk for injury Acute pain Impaired comfort: gastric distress Risk for allergic response

Risk for allergic response Explanation: When first-line treatments for gout are not successful, sometimes drugs that are more toxic may be prescribed, such as the pegloticase infusion. During the infusion the client is closely monitored for the development of adverse reactions, in particular anaphylaxis. Should an anaphylactic reaction occur, the infusion center staff members are prepared to start resuscitative measures as emergency personnel are notified. Risk for injury would be appropriate if the client is drowsy as well as pain and deformity may contribute to the risk of falling and suffering an injury. Acute pain is not a recognized nursing diagnosis for this medication. Impaired comfort: gastric discomfort may may be appropriate if the client experiences GI upset which is possible.

Assessment of a client receiving anticonvulsant therapy reveals sore throat, chills, fever, gingival bleeding, and bruising. Which nursing diagnosis would the nurse most likely prioritize? Risk for impaired skin integrity Impaired oral mucous membranes Risk for injury Risk for infection

Risk for infection Explanation: The assessment findings suggest pancytopenia, so risk for infection would be most likely. Risk for impaired skin integrity would be appropriate if the client developed a rash. Impaired oral mucous membranes would be appropriate if the client was exhibiting signs of gingival hyperplasia. Risk for injury would be appropriate if the client was experiencing drowsiness, ataxia, and vision disturbances related to the drug therapy.

The nurse is participating in the care of a client in status epilepticus who is receiving phenobarbital IV. Which nursing diagnosis should the nurse prioritize during this client's current care? Risk for injury related to seizure activity Acute confusion related to seizure activity and drug effects Disturbed thought processes related to seizure activity Disturbed sensory perception related to adverse drug effects

Risk for injury related to seizure activity Explanation: Seizures present an acute risk for injury which the nurse must prioritize when caring for the client. Cognitive and/or sensory disruptions are likely as well, but none of these presents such a high risk to the client's safety as the risk for injury.

The nurse is caring for a client who has taken hydantoins for several years. The nurse noted that the client has an unsteady gait, dizziness, nystagmus, acute confusion, a non-raised red rash on one arm, and has not been to the dentist for 2 years. Based upon the findings, what is the priority nursing diagnosis? Risk for injury related to untoward effect of hydantoin Risk for impaired oral mucous membranes related to hydantoin therapy Risk for impaired skin integrity related to adverse reactions to hydantoin Risk for infection related to immunosuppression from hydantoin

Risk for injury related to untoward effect of hydantoin Explanation: The client is exhibiting symptoms of hydantoin toxicity, including ataxia, nystagmus, dizziness, and acute onset of confusion, which are untoward effects of the drug when the drug level is in toxic range and needs to be reported immediately. The dosage of hydantoin should also be held until further orders are received from the primary care provider. The symptoms can lead to the risk for injury, and constitute the highest priority nursing diagnosis. The rash is located on one arm, and usually a drug reaction results in a generalized rash, so impaired skin integrity needs further investigation as to what may be the etiology for the rash, but not as high of a priority as the drug toxicity reaction. Risk for an infection related to immunosuppression is a risk, but there are no symptoms suggesting an onset of an infection or history of an exposure to an infection. The risk for impaired oral mucous membranes is not as high of a priority since there are no symptoms of gingival hyperplasia noted. The client does need to be seen by the dentist at least every 6 months.

A client has been prescribed a subcutaneous medication to treat rheumatoid arthritis (RA). The nurse should teach the client to do which?

Rotate sites when administering the injections.

When does a client with spinal cord injury-associated spasticity require treatment for this condition? Safety, mobility, and activities of daily living are impaired. Orthopedic procedures are indicated. Ambulation is possible only with an assistive device. Pain is present in all affected extremities.

Safety, mobility, and activities of daily living are impaired. Explanation: In clients with spinal cord injury, spasticity requires treatment when it impairs safety, mobility, and the ability to perform activities of daily living (e.g., self-care in hygiene, eating, dressing, and work or recreational activities). The treatment should be implemented before the other options occur.

The nurse is administering a bisphosphonate to a client with Paget's disease. Which finding on assessment should the nurse prioritize? Altered renal function Increased skin rashes Serum calcium levels Hematology function

Serum calcium levels Explanation: The nurse should monitor the serum levels of calcium before, during, and after bisphosphonate therapy, because bisphosphonates act primarily on the bone by inhibiting normal and abnormal bone resorption. These drugs are used cautiously in clients with renal function impairment but do not alter renal function. The nurse should monitor the client's renal function when allopurinol is administered and hematology function when methotrexate is administered.

The nurse is conducting an ongoing assessment on a client who was administered ethotoin. Which finding should the nurse prioritize? Constipation Slurred speech Diarrhea Urinary frequency

Slurred speech Explanation: The nurse should monitor the client for slurred speech, which is a sign of toxicity. Constipation, diarrhea, and urinary frequency are potential adverse reactions; however, these are not signs of ethotoin toxicity.

The nurse is conducting an ongoing assessment on a client who was administered ethotoin. Which finding should the nurse prioritize?

Slurred speech-The nurse should monitor the client for slurred speech, which is a sign of toxicity. Constipation, diarrhea, and urinary frequency are potential adverse reactions; however, these are not signs of ethotoin toxicity.

A client diagnosed with multiple sclerosis-induced spasticity asks why deep tissue massages do not relieve the associated pain. What facts about spasticity should be the basis of the nurse's response? Spasticity is caused by nerve damage in the brain and spinal cord, and it is a permanent condition. Spasticity is caused by skeletal muscle trauma and will improve only with time. Spasticity is caused by inflammation of the muscles; therefore, an anti-inflammatory drug would be more effective. Spasticity is caused by stimulation of "association areas" in the brain; therefore, only nerve blocks will be effective.

Spasticity is caused by nerve damage in the brain and spinal cord, and it is a permanent condition. Explanation: Spasticity is caused by nerve damage in the brain and spinal cord. It is a permanent condition that may be painful and disabling. None of the other options accurately provide information regarding the treatment of this condition.

The primary health care provider prescribes adalimumab. The nurse would prepare to administer this drug by which route? Orally Intramuscularly Subcutaneously Intravenously

Subcutaneously Explanation: Adalimumab, a DMARD, is administered by subcutaneous injection. Abatacept and infliximab are examples of DMARDs that are administered IV. Methotrexate, sulfasalazine, and leflunomide are examples of DMARDs that can be given orally.

A client with a history of partial seizures has been taking lamotrigine for the past several days. The client calls the clinic and reports the development of a facial and torso rash to the nurse. What is the nurse's best action? Tell the client to take the medication with a high-fat food to minimize adverse effects. Rule out any shortness of breath and inform the client that this adverse effect will resolve with time. Recommend that the client take 50 mg diphenhydramine PO and check back tomorrow. Tell the client to take no further doses and come be assessed at the clinic immediately.

Tell the client to take no further doses and come be assessed at the clinic immediately . Explanation: The nurse should inform the client to discontinue the drug and return to the clinic. Rashes associated with the use of lamotrigine can be life-threatening. The client needs to return to the clinic to be evaluated and will need a change of medication. Recommending another medication is insufficient, and is also beyond the nurse's scope. High-fat foods are of no benefit.

A client with a complex health history has developed severe spasticity. What aspect of the client's status would contraindicate the safe and effective use of dantrolene ? The client has a history of fatty liver disease. The client has type 2 diabetes and takes oral antihyperglycemics. The client has asthma and occasionally uses an inhaled corticosteroid. The client requires partial assistance with activities of daily living.

The client has a history of fatty liver disease. Explanation: Dantrolene is contraindicated in the presence of any known allergy to the drug. It is also contraindicated in the following conditions: spasticity that contributes to locomotion, upright position, or increased function, which would be lost if that spasticity was blocked; active hepatic disease (such as fatty liver disease), which might interfere with metabolism of the drug and because of known liver toxicity; and lactation because the drug may cross into breast milk and cause adverse effects in the infant. Diabetes, asthma and lack of independence do not contraindicate the use of dantrolene.

A client with a spinal cord injury is experiencing increasing muscle spasticity and the care team is considering the use of dantrolene. The nurse should identify what possible contraindication to the safe and effective use of this medication? The client has hepatitis C The client is receiving a selective serotonin reuptake inhibitor for depression The client has a sacral pressure ulcer The client's injury took place less than three months ago

The client has hepatitis C Explanation: Dantrolene is associated with possible hepatocellular injury and a concurrent diagnosis of hepatitis C would require great caution, or may contraindicate the use of dantrolene. Pressure ulcers, use of SSRIs and recent injury do not contraindicate its use.

An adult client underwent diagnostic testing after experiencing an absence seizure for the first time. What aspect of this client's health history may result in impaired drug excretion? The client has recently been diagnosed with diabetic nephropathy The client takes a beta-blocker for the treatment of hypertension The client is morbidly obese The client has a history of adhering poorly to prescribed treatment

The client has recently been diagnosed with diabetic nephropathy Explanation: Impaired renal function will reduce drug excretion. A lack of adherence must be addressed but this does not affect excretion. Similarly, obesity and the use of a beta-blocker will not inhibit excretion.

When reviewing the medical record of a client who is to receive dantrolene, which condition would alert the nurse to the need for close monitoring? The client has respiratory depression from opioid analgesia The client received a blood transfusion less than 24 hours ago The client is 67 years old The client has an infected surgical wound

The client has respiratory depression from opioid analgesia Clients with respiratory depression need to be monitored closely because the use of dantrolene can increase muscular weakness and exacerbate the respiratory depression. None of the other listed client factors would necessarily require increased monitoring.

A client is prescribed topiramate. The nurse cautions the client to avoid the intake of alcohol for which reason? The client is at risk for increased CNS depression. A disulfiram-like reaction can occur. The client's risk for dependence is increased. The combination may lead to a hypertensive crisis.

The client is at risk for increased CNS depression. Explanation: Combining alcohol with topiramate increases the client's risk for CNS depression. Topiramate combined with alcohol does not result in a disulfiram-like reaction. Drug dependency is not associated with the use of topiramate and alcohol. The combination of topiramate and alcohol is not associated with the development of a hypertensive crisis.

A client has a new diagnosis of a seizure disorder. What aspect of this client's health status would contraindicate the use of carbamazepine? The client has an allergy to sulfonamides. The client is 17 years old. The client's most recent blood work reveals pancytopenia. The client has type 2 diabetes, controlled through diet.

The client's most recent blood work reveals pancytopenia. Explanation: Bone marrow suppression would be considered a contraindication to administration of carbamazepine therapy. Contraindications to the use of Tegretol do not include an allergy to sulfonamides, or diabetes. The drug could be safely administered to a 17 year-old client.

A 60 year-old female client has multiple sclerosis accompanied by muscle spasticity. The client has responded well to dantrolene 200 mg PO daily in divided doses. What assessment finding should the nurse prioritize for communication to the client's provider? The client's most recent laboratory results show an upward trend in AST and ALT levels The client has gained 1.5 lbs. over the past 48 hours The client is reluctant to participate in physical therapy this morning, when she is normally highly motivated The client tells the nurse, "I'm struggling to come to grips with the fact that I'm never going to recover."

The client's most recent laboratory results show an upward trend in AST and ALT levels Explanation: An increase in liver enzyme levels could indicate the onset of hepatocellular damage, which is a high risk in female clients of this age. The provider must be made aware of this immediately. The client's lack of motivation and apparent despair should be addressed by the nurse, but may be able to be addressed independently by the nurse. As well, these issues are less time-dependent than a decrease in liver function. A weight gain of 1.5 lbs. over 48 hours is not outside the range of usual fluctuations in fluid balance.

A client with a family history of malignant hyperthermia is to have bowel resection surgery and has been prescribed dantrolene 2.5 mg/kg IV one hour before surgery. The client weighs 185 lbs. The nurse reconstitutes dantrolene to a concentration of 50 mg/mL and should add how many mL to the IV minibag? Provide your answer to one decimal place.

The client's weight must first be converted to kg by dividing by 2.2. This yields 84.1 kg. The client is to receive 2.5 mg/kg and 2.5 X 84.1 = 210.25 mg. Dividing 210.25 by 50 mg/mL = 4.2 mL.

Which of the following drugs are approved for use in MS clients with severe spasticity? Metaxalone and Carisoprodol Dantrolene and Carisoprodol Tizanidine and Dantrolene Tizanidine and Baclofen

Tizanidine and Baclofen Explanation: Baclofen and tizanidine are approved for spasticity in people with MS. The two drugs are similarly effective, but tizanidine may cause fewer adverse effects.

When assessing a client for possible adverse effects of direct-acting cholinergic agents, what might the nurse find?

Urinary urgency Explanation: Adverse effects associated with direct-acting cholinergic agents include urinary urgency, bradycardia, hypotension, and diarrhea.

A client is receiving hydroxychloroquine for a musculoskeletal disorder. Which adverse reactions are irreversible and needs to be reported immediately? Easy bruising Skin rash Fever Visual changes

Visual changes Explanation: The nurse needs to report visual changes in a client receiving hydroxychloroquine, because irreversible retinal damage may occur. Although easy bruising, skin rash, and fever are adverse reactions of the drug, they are not irreversible.

The client has been diagnosed with myasthenia gravis. The nurse knows that which is a symptom of myasthenia gravis?

Weak and easily fatigued muscles Explanation: Myasthenia gravis is a neuromuscular condition characterized by weakness and easily fatigued muscles. Inability to urinate and stand are not symptoms of myasthenia gravis. Daytime sleepiness is not a symptom of myasthenia gravis.

The health care provider orders short-term skeletal muscle relaxants for an 11-year-old client. The nurse is responsible for the family education plan and teaches the parents that the medications should be used only under which condition? When the client needs to be alert during pain management During school hours to increase alertness and management of spasms When close supervision is available for monitoring drug effects When the spasms cause uncontrolled pain

When close supervision is available for monitoring drug effects Explanation: For most of the skeletal muscle relaxants, safety and effectiveness for use in children 12 years of age and younger have not been established. The drugs should be used only when clearly indicated, for short periods, when close supervision is available for monitoring drug effects (especially sedation), and when mobility and alertness are not require

The nurse is preparing to administer the next dose of methsuximide to a client experiencing partial seizures. The client remarks about hoping their stomach will do better this time. Which suggestion from the nurse will best assist this client when administering this next dose? Daily at bedtime Immediately before eating a meal With some food or milk First thing in the morning on arising

With some food or milk Explanation: If the client experiences GI upset after succinimide administration, the nurse should instruct the client to take the drug with food or milk. The nurse need not instruct the client to take the drug once only at bedtime, before meals, or immediately on arising in the morning.

A client is experiencing absence seizures and has been prescribed a succinimide. What prescription is most likely? Zarontin Depakene Dilantin Valium

Zarontin Explanation: Ethosuximide (Zarontin) is considered the drug of choice for treating absences seizures and is in the succinide family. The other listed drugs are not succinides.

The nurse is caring for four clients. Which client would have the highest risk for hepatotoxicity from dantrolene? a man who is taking a cardiac glycoside. a man who it taking an antipsychotic drug a woman who is on hormone replacement therapy a woman who is taking a loop diuretic

a woman who is on hormone replacement therapy Explanation: If dantrolene is combined with estrogen, the incidence of hepatocellular toxicity is increased. This combination should be avoided. Nothing indicates that clients taking a cardiac glycoside, an antipsychotic drug, and an antihypertensive would have serious adverse effects when combined with dantrolene therapy.

The nurse is educating a client who has been prescribed atropine, an anticholinergic drug. Which instructions should the nurse provide to the client? a. "Do not drive or operate machinery." b. "If you experience eye pain, contact the health care provider about increasing the dosage." c. "Increase your fluid intake and activity level." d. "Take long, hot showers frequently to relieve uncomfortable side effects."

a. "Do not drive or operate machinery." Effects of anticholinergic drugs such as atropine include blurred vision and impaired ability to sweat. Clients should be cautioned to avoid activities that may be made dangerous by blurred vision; this may include driving and operating machinery. To avoid overheating, clients should avoid strenuous activity and take other measures to stay cool (e.g., frequent cool baths). Clients experiencing eye pain may have undiagnosed glaucoma; they should stop taking the medications and contact their health care providers.

A male client is diagnosed with Parkinson's disease. The nurse visits him 1 week after hospitalization to assess his progress and medication regimen compliance. He states that there is no significant improvement in his symptoms. What is the best response by the nurse? a. "Noticeable improvement may not occur for several weeks after the initial drug dose." b. "Noticeable improvement usually occurs 7 to 10 days after the initial drug dose." c. "You should have noticed improvement by now; I will notify your health care provider." d. "Are you sure that you are taking the medication as ordered?"

a. "Noticeable improvement may not occur for several weeks after the initial drug dose." The home care nurse can help clients and caregivers understand that the purpose of drug therapy for Parkinson's disease is to control symptoms and that noticeable improvement may not occur for several weeks.

The pharmacology instructor is discussing age-related susceptibility to adverse effects of cholinergic drugs. Which statement could the instructor make to accurately describe the influence of age? a. "Physiologic changes resulting from the normal aging process place older adults at higher risk for adverse effects." b. "Older adults are at higher risk for adverse effects because they receive higher doses." c. "All age groups are equally susceptible to adverse effects." d. "Children are more susceptible to adverse effects because they receive more potent drugs than adults."

a. "Physiologic changes resulting from the normal aging process place older adults at higher risk for adverse effects." Age-related physiologic changes (e.g., to metabolism and excretion) and a higher likelihood of superimposed pathologic conditions (e.g., renal dysfunction) place older adults at increased risk for adverse effects of cholinergic drugs.

An 82-year-old client is newly diagnosed with Alzheimer disease. The client's adult child asks the nurse how the medication is going to help. What should the nurse explain in layman's terms? a. "The drugs work by increasing acetylcholine (ACh) levels in the brain and slowing the progression of the disease." b. "The drugs work by crossing the blood-brain barrier and cure the disease in the brain." c. "The drugs work by increasing ACh levels in the brain and reverse the progression of the disease." d. "The drugs work by crossing the blood-brain barrier and decreasing ACh levels in the neuromuscular junctions."

a. "The drugs work by increasing acetylcholine (ACh) levels in the brain and slowing the progression of the disease." Acetylcholinesterase inhibitors that cross the blood-brain barrier are used to manage Alzheimer disease by increasing ACh levels in the brain and slowing the progression of the disease. Decreasing ACh levels in the brain does not slow the progression, reverse the progression, or cure the disease. Decreasing ACh levels at the neuromuscular junctions has no effect on Alzheimer disease.

A nurse caring for a client receiving bethanechol for urinary retention should advise the client that voiding usually occurs how long after oral administration? a. 30 to 90 minutes b. 5 to 15 minutes c. 2 to 4 hours d. 12 to 24 hours

a. 30 to 90 minutes A nurse caring for a client receiving bethanechol (Urecholine) for urinary retention should advise the client that voiding usually occurs 30 to 90 minutes after an oral dose is administered. The time frame of 5 to 15 minutes does not allow enough time for the medication to be absorbed and distributed. The time frame of 2 to 4 hours indicates that the medication will not work and the time frame of 12 to 24 hours indicates that the client will have bladder distention and that an insertion of a catheter may need to be placed to relieve the pressure exerted on the bladder.

A nurse has administered a drug that will have a therapeutic effect on the client's parasympathetic nervous system. The nurse understands the drug will primarily affect what neurotransmitter? a. Acetylcholine b. Norepinephrine c. Dopamine d. Serotonin

a. Acetylcholine Acetylcholine (ACh) is the neurotransmitter that produces parasympathetic effects. Serotonin, dopamine and norepinephrine do not.

A nursing student is conducting a class presentation about cholinesterase inhibitors. Which would the student identify as the primary indication for use? a. Alzheimer disease b. Myasthenia gravis c. Glaucoma d. Urinary retention

a. Alzheimer disease Cholinesterase inhibitors strengthen neurotransmission and are used for the treatment of Alzheimer disease primarily, and at times for dementia of Parkinson disease. Myasthenia gravis is a chronic autoimmune neuromuscular disorder characterized by weakness and muscle fatigue while glaucoma is a group of diseases that damage the eye's optic nerve leading to vision loss and blindness, neither are treated with cholinesterase inhibitors. Urinary retention is a common side effect of some drugs but with the use of cholinesterase inhibitors the most common side effects are anorexia, nausea and vomiting, diarrhea, dizziness, and headache.

The nurse is providing education to a client who has been prescribed an anticholinesterase drug. What caution would the nurse include in the teaching? a. Anticholinesterase drugs should be taken with food or milk to decrease the risk of gastric distress and ulceration. b. Anticholinesterase drugs should be taken on an empty stomach to decrease the risk of gastric distress and ulceration. c. Anticholinesterase drugs should be taken at bedtime to prevent nausea. d. Anticholinesterase drugs need to be started at a very high dose and titrated to effect.

a. Anticholinesterase drugs should be taken with food or milk to decrease the risk of gastric distress and ulceration. All anticholinesterase drugs should be taken with food or milk to decrease the risk of gastric distress and ulceration. These drugs are taken at bedtime to avoid the risks associated with dizziness and vertigo, not to prevent nausea. These drugs are not started at very high doses, though doses may be increased after several weeks if initial dosing doesn't achieve the desired effect.

A client experiences an overdose of a cholinergic drug. Which medication would the nurse anticipate that the client will receive as a reversal agent? a. Atropine b. Dopamine c. Epinephrine d. Norepinephrine

a. Atropine Atropine is considered an antidote for a cholinergic drug overdose. Dopamine is a catecholamine neurotransmitter that is given to increase the contraction of the heart in shock. Epinephrine is a hormone released in the presence of stress and is given in the case of allergic reaction and norepinephrine is a neurotransmitter from the autonomic nervous system, that constricts blood vessels, raising blood pressure.

Which would be important to perform for a client who has received an anticholinergic agent? (Select all that apply.) a. Auscultating bowel sounds b. Monitoring urine output c. Monitoring heart rate d. Administering an antidiarrheal agent e. Assessing deep tendon reflexes

a. Auscultating bowel sounds b. Monitoring urine output c. Monitoring heart rate Anticholinergic agents can slow GI activity leading to paralytic ileus. Urinary retention is possible, so it is important to monitor the patient's urine output. Tachycardia is possible, so monitoring the heart rate would be important. The client would most likely experience constipation, not diarrhea. Anticholinergics do not affect deep tendon reflexes, so assessment is not necessary.

A nurse is reviewing the medication history of several clients. One of the clients is receiving a direct-acting cholinergic drug. The nurse would most likely identify which drug? a. Bethanechol b. Pyridostigmine c. Guanidine d. Ambenonium

a. Bethanechol Bethanechol is an example of a direct-acting cholinergic that acts like the neurotransmitter acetylcholine. Pyridostigmine, guanidine, and ambenonium are indirect-acting muscle stimulants.

Administration of which classes of medications can decrease secretions of the upper respiratory tract? a. Cholinergic blocking drug b. Opioids c. Antianxiolytic d. Antiemetics

a. Cholinergic blocking drug Cholinergic antagonists are the class of medications used to decrease secretions of the upper respiratory tract. Antianxiolytics decrease preoperative anxiety. Opioids decrease both anxiety and pain preoperatively. Antiemetics are used to decrease nausea after surgery.

A nurse is caring for 70-year-old patient who is undergoing anticholinergic drug therapy. The nurse should assess for which condition when caring for this elderly patient? a. Confusion and disorientation b. Choreiform movements c. Suicidal tendencies d. Psychotic episodes

a. Confusion and disorientation The nurse should assess for confusion and disorientation when caring for this elderly patient undergoing anticholinergic drug therapy. Individuals older than 60 years frequently develop increased sensitivity to anticholinergic drugs and require careful monitoring. Lower doses may also be required in such cases. Choreiform movements, suicidal tendencies, and psychotic episodes are serious adverse reactions associated with the use of levodopa, which is a dopaminergic drug.

Which would a nurse least likely expect to assess in a client experiencing mild atropine toxicity? a. Cough b. Dry mouth c. Profuse sweating d. Increasing of heart rate

a. Cough Cough is not associated with mild atropine toxicity. Dry mouth is associated with mild atropine toxicity. Inhibition of sweating is associated with mild atropine toxicity not profuse sweating. Slight cardiac slowing is associated with mild atropine toxicity not increased heart rate.

Which of the following cholinergic-blocking drugs only affect the muscarinic receptors? Select all that apply: a. Darifenacin (Enablex) b. Oxybutynin (Ditropan) c. Benztropine (Cogentin) d. Biperiden (Akineton) e. Tolterodine (Detrol)

a. Darifenacin (Enablex) b. Oxybutynin (Ditropan) e. Tolterodine (Detrol) Antispasmodic cholinergic-blocking drugs, like darifenacin , oxybutynin (Ditropan), and tolterodine, only affect muscarinic receptors in the parasympathetic nervous system and have no affect on nicotinic receptors.

After administering an antiparkinson drug to a client, the nurse assesses for the drug's effectiveness. The nurse determines that the drug is effective based on assessment of which of the following? Select all that apply. a. Decrease in muscle rigidity b. Improved gait c. Reduction in tremors d. Greater slowing of movement e. Reduced dry mouth

a. Decrease in muscle rigidity b. Improved gait c. Reduction in tremors The drug is intended to address the signs and symptoms of Parkinson disease, such as muscle rigidity, shuffling gait, tremors, and slow movements. Therefore, a decrease in muscle rigidity, improved gait, and reduction in tremors would indicate drug effectiveness. Greater slowing of movement would indicate that the drug is not effective. Dry mouth is an adverse reaction and not considered an indicator of drug effectiveness.

A nursing mother is being prepared for an eye condition which requires a prescription for atropine. What nursing intervention should the nurse implement to assure safe and effective atropine therapy? a. Discuss an alternative feeding method until the atropine is no longer prescribed b. Discuss pumping and then warming the breast milk before bottle feeding the baby c. Discuss feeding the baby prior to taking the prescribed atropine d. Discuss the mother's need to be well hydrated to minimize the amount to atropine in the breastmilk

a. Discuss an alternative feeding method until the atropine is no longer prescribed Because of the potential for serious adverse effects on the baby, nursing mothers should be assisted in finding another method of feeding the baby if an anticholinergic drug is needed. None of the other options effectively manage the safety risk to the infant.

A nurse is caring for a patient with urinary retention. Which of the following conditions should the nurse notify the primary healthcare provider as part of the ongoing assessment? a. Failure to void after drug administration b. Frequent vomiting after drug administration c. Increase in abdominal pain after drug administration d. Occurrence of blood in urine after drug administration

a. Failure to void after drug administration The nurse should notify the primary healthcare provider that the patient has failed to void after drug administration. Frequent vomiting, increase in abdominal pain and occurrence of blood in urine are not usually observed for urinary retention treatment by cholinergic therapy.

When describing the various anticholinergic agents to a group of students, the instructor would identify which agent as being available in sublingual form? a. Hyoscyamine b. Scopolamine c. Propantheline d. Tiotropium

a. Hyoscyamine Hyoscyamine is available in oral or sublingual form and for subcutaneous, IM, or IV use. Scopolamine is available for subcutaneous or IM injection, in ophthalmic form, and as a transdermal system. Propantheline is available for oral use only. Tiotropium is available for inhalation.

After teaching a group of students about direct acting cholinergic agonists, the instructor determines that additional teaching is necessary when the students identify what as an adverse effect? a. Hypertension b. Bradycardia c. Diarrhea d. Urinary urgency

a. Hypertension Direct acting cholinergic agonists can cause bradycardia, hypotension, diarrhea, and urinary urgency.

The pharmacology instructor is discussing the drug therapy for Alzheimer's disease with a group of nursing students and states that indirect-acting cholinergic drugs do what to improve symptoms? a. Improve cholinergic neurotransmission to the brain b. Improve anticholinergic neurotransmission to the brain c. Improve medication absorption through the blood-brain barrier d. Cause vasodilation of the cerebral arteries

a. Improve cholinergic neurotransmission to the brain Indirect-acting cholinergic medications for Alzheimer's disease are widely distributed, including to the central nervous system. Thus, indirect-acting cholinergic drugs are able to improve cholinergic neurotransmission in the brain.

A patient with dementia of Alzheimer's disease is administered donepezil HCL. What are the actions associated with cholinesterase inhibitors? a. Increase in the level of acetylcholine in CNS b. Decrease in the level of neurotoxins in the brain c. Increase in the level of cholinesterase in the blood d. Increase in the level of adenosine tri-phosphate in the blood

a. Increase in the level of acetylcholine in CNS Administration of cholinesterase inhibitors results in an increase in the level of acetylcholine in CNS. Administration of cholinesterase inhibitors does not decrease the neurotoxins in the brain or increase the level of adenosine tri-phosphate in the blood. Cholinesterase inhibitors do not increase the level of cholinesterase in the blood.

The nurse is caring for a client with Alzheimer's disease who has been prescribed a cholinesterase inhibitor. What symptom of the disorder does the nurse expect the medication is intended to treat? a. Mild to moderate dementia b. Severe dementia c. Wandering and aggression d. Irritability

a. Mild to moderate dementia Cholinesterase inhibitors are used to treat mild to moderate dementia, not other symptoms of AD such as wandering, irritability, or aggression. The anticipated therapeutic effects are improved memory and reduction of dementia.

When describing the action of direct-acting cholinergic agonists, which receptors would the nurse identify as being stimulated? a. Muscarinic b. Nicotinic c. Alpha d. Beta

a. Muscarinic Direct-acting cholinergic agonists usually stimulate muscarinic receptors within the parasympathetic nervous system. Alpha and beta receptors are found in the sympathetic nervous system and are affected by adrenergic agents.

After teaching a group of students about indirect acting cholinergic agonists used to treat myasthenia gravis, the instructor determines that the teaching was effective when the students identify which agent? a. Neostigmine b. Galantamine c. Rivastigmine d. Tacrine

a. Neostigmine Neostigmine is used to treat myasthenia gravis. Galantamine is used to treat Alzheimer's disease. Rivastigmine is used to treat Alzheimer's disease. Tacrine is used to treat myasthenia gravis.

Which of the following places the pt at risk for toxicity following the administration of phenytoin? a) the pt skips a dose of phenytoin b) the pt takes a different brand of phenytoin c) the pt switches to a different antiepileptic agent d) the pt receives phenytoin in an enteral feeding

b) the pt takes a different brand of phenytoin

The stimulation of which receptors in the parasympathetic nervous system stimulates skeletal muscle? a. Nicotinic receptors b. Muscarinic receptors c. Alpha receptors d. Beta receptors

a. Nicotinic receptors The stimulation of nicotinic receptors in the parasympathetic nervous system stimulates skeletal muscle. The stimulation of muscarinic receptors stimulates smooth muscle. Alpha receptors are part of the sympathetic nervous system. Beta receptors are part of the sympathetic nervous system.

A client is prescribed donepezil. The nurse would expect to administer this drug at which frequency? a. Once a day b. Twice a day c. Three times a day d. Four times a day

a. Once a day Donepezil has a 70-hour half-life and is usually given in a once-a-day dosing. Galantamine and rivastigmine are usually taken twice a day. Tacrine must be taken four times a day.

A client is prescribed benztropine. The nurse would anticipate administering this drug by which route? a. Oral b. Subcutaneous c. Intramuscular d. Intravenous

a. Oral Benztropine is administered orally.

What drug is prescribed for mushroom poisoning? a. atropine b. pralidoxime c. bethanechol d. neostigmine

a. atropine Accidental or intentional ingestion of these mushrooms results in intense cholinergic stimulation and is potentially fatal. Atropine is the specific antidote for mushroom poisoning.

The nurse is reviewing the medication history of a client who is receiving drug therapy for Alzheimer's disease. What medication should the nurse question? a. atropine b. donepezil c. memantine d. rivastigmine

a. atropine Medications that may be beneficial in slowing the progression of Alzheimer's disease include rivastigmine, donepezil, and memantine. Atropine is not used for this purpose.

What are the classic symptoms associated with Parkinson's disease? Select all that apply. a. bradykinesia b. muscle rigidity c. urinary retention d. resting tremors e. worsening eyesight

a. bradykinesia b. muscle rigidity d. resting tremors Classic symptoms of Parkinson's disease include resting tremor, bradykinesia, rigidity, and postural instability.

A client has been prescribed benztropine as drug therapy for Parkinson disease. What assessment finding would suggest a therapeutic effect to the nurse? a. decreased rigidity and tremors b. increased level of consciousness c. increased motivation and muscle strength d. absence of seizure activity

a. decreased rigidity and tremors Benztropine should cause a decrease in rigidity and tremors. It does not affect LOC, motivation, strength, or seizure risk.

Common side effects of anticholinergics include: a. dry mouth, urinary retention, constipation, and increased pulse rate. b. blurred vision, decreased liver enzymes, increased B/P, and papillary constriction. c. decreased GI absorption, nausea, heartburn, and hypotension. d. skin rashes, decreased urinary output, constipation, and increased WBCs.

a. dry mouth, urinary retention, constipation, and increased pulse rate. Anticholinergic drugs may cause blurred vision, dry mouth, tachycardia, and urinary retention. They also decrease sweating and may cause fever or heatstroke. Fever may occur in any age group, but heatstroke is more likely to occur in older adults, especially with cardiovascular disease, strenuous activity, and high environmental temperatures. When centrally active anticholinergics are given for Parkinson's disease, agitation, mental confusion, hallucinations, and psychosis may occur.

The nurse is caring for a client who is receiving neostigmine for the treatment of myasthenia gravis. The nurse is aware that a potential complication of the treatment is cholinergic crisis, which is characterized by which event? a. excessive stimulation of the parasympathetic nervous system b. excessive stimulation of the sympathetic nervous system c. hypertension and respiratory failure d. stronger skeletal muscle contractions

a. excessive stimulation of the parasympathetic nervous system A cholinergic crisis is characterized by excessive stimulation of the parasympathetic nervous system. If early symptoms are not treated, hypotension and respiratory failure may occur. At high doses, anticholinesterase drugs weaken, rather than strengthen, skeletal muscle contraction because excessive amounts of acetylcholine accumulate at motor endplates and reduce nerve impulse transmission to muscle tissue.

Because of the systemic effects of anticholinergic drugs, patients using these drugs are more susceptible to: a. heat stroke related to decreased perspiration. b. diarrhea related to increased peristalsis. c. urinary frequency related to CNS effects. d. hypotension related to increased sympathetic response.

a. heat stroke related to decreased perspiration. By blocking cholinergic receptors, anticholinergic drugs reduce sweating, which decreases heat loss and increases the risk of hyperthermia and heat stroke.

What information about the use should the nurse provide a client preoperatively prescribed atropine? a. it prevents complications such as bradycardia and hypotension b. atropine is given preoperatively to assist in thinning of respiratory secretions c. it sedates the client, enhancing anesthesia. d. atropine is given to prevent nausea and vomiting

a. it prevents complications such as bradycardia and hypotension

For a client diagnosed with both Parkinson's and narrow-angle glaucoma, what medication should cause the nurse concern? a. levodopa b. gabapentin c. phenytoin d. amantadine

a. levodopa

The nurse is caring for a postoperative client who is experiencing abdominal distension related to a paralytic ileus. What drug would the nurse expect to be prescribed? a. neostigmine b. pyridostigmine c. edrophonium d. bethanechol

a. neostigmine Bethanechol is used to treat urinary retention due to urinary bladder atony and postoperative abdominal distention due to paralytic ileus. Neostigmine is the prototype anticholinesterase agent. It is used for long-term treatment of myasthenia gravis and as an antidote for tubocurarine and other nondepolarizing skeletal muscle relaxants used in surgery as well as paralytic ileus. Pyridostigmine is similar to neostigmine in actions, uses, and adverse effects. Edrophonium is a short-acting cholinergic drug used to diagnose myasthenia gravis, to differentiate between myasthenic crisis and cholinergic crisis, and to reverse the neuromuscular blockade produced by nondepolarizing skeletal muscle relaxants.

The nurse is providing education to a client who has been prescribed donepezil. The nurse should instruct the client to take the medication according to what schedule? a. once daily at bedtime b. each morning with food c. three times a day with meals d. every 2nd day on an empty stomach

a. once daily at bedtime Administration of donepezil should occur at bedtime each day.

While being intubated, a client with a head injury experienced bradycardia. Atropine was consequently administered. What assessment should be postponed until all atropine is excreted and no longer exerting an effect? a. pupil response b. electroencephalogram c. brainstem reflexes d. computed tomography of the brain

a. pupil response One test for neurological function is to shine a light in the client's eyes to test pupil reaction to light. Because this client has received atropine, pupils will be dilated and will not react normally to light. This could be mistaken as an indication of brain death if the nurse did not know atropine had been administered. This test will be postponed until the pupils are no longer dilated by the medication. The other tests would not have to be postponed because of atropine.

The nurse is speaking to a group at the senior citizen's center about Parkinson disease. What should the nurse explain is importance to avoiding when taking an anticholinergic medication? a. strenuous exercise in high environmental temperatures b. salt substitutes and foods high in potassium c. foods high in vitamin K such as dark green, leafy vegetables d. over-the-counter medications containing acetaminophen

a. strenuous exercise in high environmental temperatures Anticholinergic drugs decrease sweating. As a result, the body is not as effective at reducing internal temperature as this could result in fever and heatstroke. Elderly people taking anticholinergic drugs do not need to avoid potassium, foods high in vitamin K, or acetaminophen.

What is a common side effect of anticholinergic drug therapy? Select all that apply. a. urinary retention b. skin rash c. blurred vision d. papillary constriction e. dry mouth

a. urinary retention c. blurred vision e. dry mouth

In a person being treated for a diagnosed seizure disorder, what is the most common cause of status epilepticus? abusing drugs and/or alcohol experiencing a brain injury abruptly stopping the antiseizure medications hypoxia-inducing experience

abruptly stopping the antiseizure medications Explanation: In a person taking medications for a diagnosed seizure disorder, the most common cause of status epilepticus is abruptly stopping AEDs. In other clients, regardless of whether they have a diagnosed seizure disorder, causes of status epilepticus include brain trauma or tumors, systemic or central nervous system (CNS) infections, alcohol withdrawal, and overdoses of drugs (e.g., cocaine, theophylline).

A client with a recent history of seizures has been prescribed carbamazepine. The nurse should recognize that this medication will achieve a therapeutic effect by what means? depressing conduction in the brainstem and cortex and slowing the reuptake of dopamine reducing electrical activity in the thalamus and hypothalamus altering the permeability of neuron cell membranes affecting gamma-aminobutyric acid (GABA) levels and blocking sodium channels

affecting gamma-aminobutyric acid (GABA) levels and blocking sodium channels Explanation: Carbamazepine affects GABA activity and inhibits sodium channels to stop action potentials. Succinimides reduce electrical activity. Acetazolamides reduce electrical activity and alter sodium and calcium channels. Barbiturates depress conduction in the brainstem and the cortex but do not affect dopamine reuptake.

A nurse is caring for a patient who is starting gabapentin therapy. During the initial assessment the patient informs the nurse that he takes antacids every day. The nurse will stress to the patient that gabapentin should be taken

at least 2 hours after administering antacids.-The only important drug interaction of gabapentin is with antacids. Antacids reduce the bioavailability of the gabapentin by 20%. To avoid this interaction, gabapentin should be administered at least 2 hours after taking antacids.

What drug is prescribed for mushroom poisoning?

atropine Explanation: Accidental or intentional ingestion of these mushrooms results in intense cholinergic stimulation and is potentially fatal. Atropine is the specific antidote for mushroom poisoning.

A client in the emergency department has been diagnosed with mushroom poisoning. What drug does the nurse expect the health care provider to prescribe?

atropine Explanation: Accidental or intentional ingestion of certain mushrooms results in intense cholinergic stimulation and is potentially fatal. Atropine is the specific antidote for mushroom poisoning. Pralidoxime is used in the treatment of nerve gas exposure. Bethanechol is administered during the acute postoperative and postpartum periods for the treatment of nonobstructed urinary retention and neurogenic atony of the bladder muscle. Neostigmine is used in the diagnosis and treatment of myasthenia gravis

A client reports sensing an unusual smell just prior to experiencing a tonic-clonic seizure. What term is used to describe this event? spasticity epilepsy aura spasm

aura Explanation: Tonic-clonic seizures are sometimes preceded by an aura—a brief warning, such as a flash of light or a specific sound or smell. None of the other options accurately describe this event.

The nurse is providing education to a client who has been recently diagnosed with myasthenia gravis. During teaching, the nurse would describe it as what type of disorder?

autoimmune Explanation: Myasthenia gravis is a neuromuscular disease leading to fluctuating muscle weakness and fatigability. It is an autoimmune disorder, in which weakness is caused by circulating antibodies that block acetylcholine receptors at the postsynaptic neuromuscular junction, inhibiting the stimulative effect of the neurotransmitter acetylcholine. None of the other options describe the disorder.

A client with muscle spasticity has been prescribed baclofen. In order to promote the safe use of this medication, the nurse should encourage the client to: avoid eating grapefruit or drinking grapefruit juice until treatment is discontinued. increase fluid intake and self-monitor for decreased urine output . avoid drinking alcohol for the duration of treatment. increase intake of foods that are high in potassium.

avoid drinking alcohol for the duration of treatment. Explanation: The CNS depressant effects of baclofen can be dangerously exacerbated by alcohol, which should be avoided. Grapefruit is not contraindicated and there is no reason to increase potassium or fluid intake while taking baclofen

An older client is taking an anticholinergic agent. After teaching the client about the drug, which client statement indicates the need for additional teaching? a. "I should make sure that I drink plenty of fluids." b. "I need to exercise frequently outside in the warm weather." c. "I should avoid driving if I feel light-headed or dizzy." d. "I should eat plenty of fiber to prevent constipation."

b. "I need to exercise frequently outside in the warm weather." The client should avoid temperature extremes and exertion in warm temperatures because of possible heat intolerance, which could be more severe in older clients. Drinking fluids is important to maintain hydration and prevent heat intolerance. Avoiding driving is an appropriate safety measure. Constipation may occur with an anticholinergic; therefore, increased fiber intake would be appropriate.

Some anticholinergic drugs have the same clinical indications in the pediatric population as in the adult population. Which statement accurately describes a difference between the populations related to the use of anticholinergics? a. Anticholinergic drugs create different adverse effects in children. b. Adverse effects tend to be more severe in children. c. Anticholinergic drugs are more effective in the pediatric population. d. Adverse effects tend to be less severe in children.

b. Adverse effects tend to be more severe in children. Anticholinergic drugs have essentially the same intended effects and adverse effects in the pediatric population as in the adult population. However, children are especially sensitive to these drugs, so the adverse effects may be more severe in that population.

The nursing instructor is teaching the students about degenerative diseases that affect both the musculoskeletal and neurologic systems. The student correctly identifies the disease that is a progressive deterioration of emotional, physical, and cognitive abilities as which? a. osteoarthritis. b. Alzheimer's c. rheumatoid arthritis. d. dementia.

b. Alzheimer's Alzheimer's disease is a progressive deterioration of emotional, physical, and cognitive abilities. Osteoarthritis and rheumatoid arthritis do not normally affect the cognitive function of a client. Dementia is a group of symptoms affecting social and intellectual abilities but not physical abilities.

The health care provider is preparing to administer edrophonium to confirm myasthenia gravis in a 47-year-old male client. What other drug must be available when edrophonium is administered? a. Pyridostigmine b. Atropine c. Neostigmine d. Rivastigmine

b. Atropine Edrophonium is a cholinergic agonist administered by a provider to diagnose myasthenia gravis. Use of this drug requires that an antidote, typically atropine, be on hand in case of cholinergic overdose.

What would be considered a peripheral anticholinergic effect of anticholinergic drug therapy? a. Delirium b. Blurred vision c. Agitation d. Memory loss

b. Blurred vision Blurred vision is considered a peripheral anticholinergic effect. Delirium, agitation, and memory loss are considered central effects affecting the CNS.

The client is to receive an IV cholinergic medication. When the nurse administers the drug what reaction will the nurse anticipate? a. Increase in heart rate b. Decrease in heart rate c. Increase in anxiety d. Decrease in anxiety

b. Decrease in heart rate Cholinergic drugs decrease, not increase, heart rate. They do not usually have any effect on anxiety.

The nurse is caring for a client in the emergency department who has a history of myasthenia gravis. The client has symptoms that could be interpreted as either myasthenic crisis or cholinergic crisis. The nurse is aware that the health care provider will order what medication to help determine the type of crisis the client is experiencing? a. Donepezil b. Edrophonium c. Galantamine hydrobromide d. Memantine hydrochloride

b. Edrophonium Edrophonium is the drug that is used to help determine if a client is experiencing a myasthenic or a cholinergic crisis.

The nurse understands that older clients taking anticholinergic drugs are more likely to have all of the following symptoms EXCEPT: a. Blurred vision. b. Hypertension. c. Constipation. d. Confusion.

b. Hypertension. Older adults are especially likely to have significant adverse reactions because of slowed drug metabolism and the frequent presence of several disease processes. Some common adverse effects are blurred vision, confusion, constipation, and confusion or other psychotic symptoms. These are most likely to occur with the centrally active anticholinergics given for Parkinson's disease or drug-induced extrapyramidal effects, such as trihexyphenidyl or benztropine. They are not typically the causative agent for hypertension in routine medication treatment.

Which physiologic effect would be related to the use of cholinergic agents? a. Pupil dilation b. Increased salivation c. Increased heart rate d. Decreased bladder muscle tone

b. Increased salivation Cholinergic agents cause increased salivation, pupil constriction, decreased heart rate, and increased bladder muscle tone.

A nurse is caring for a patient with urinary retention. The patient has been prescribed Urecholine. For which category of patients should the nurse take precautions when administrating Urecholine? a. Patients with Raynaud's disease b. Patients with bradycardia c. Patients with recent myocardial infarction d. Patients with hyperthyroidism

b. Patients with bradycardia The nurse should administer Urecholine cautiously in patients with bradycardia, hypertension, epilepsy, cardiac arrhythmias, recent coronary occlusion, and megacolon. The nurse need not take precautions for the patients with Raynaud's disease, recent myocardial infarction, and hyperthyroidism when administrating Urecholine.

Which agent would a nurse expect to administer transdermally? a. Atropine b. Scopolamine c. Dicyclomine d. Propantheline

b. Scopolamine Only scopolamine is available as a transdermal system.

The nursing instructor is teaching a student about the drugs used for Alzheimer's disease. Even though cholinesterase inhibitors do not cure the disease, the instructor informs the student that they do help to slow the progression. The instructor then asks the student, "When a drug is stopped due to side effects, what happens to the client?" The student's best response would be: a. The side effects will worsen. b. The client loses any benefit they have received from the drug. c. The client will deteriorate at a faster rate. d. The client will show no changes.

b. The client loses any benefit they have received from the drug. When a cholinesterase inhibitor has to be stopped due to adverse effects then within 6 weeks of the discontinuation of the therapy, the client will lose any benefit they have received from the drugs.

The nurse is preparing a client for pacemaker surgery. The health care provider orders atropine to be given 30 minutes before the client is taken to the operating room. The nurse knows this medication is ordered for what reason? a. To increase the heart rate to prevent complication prior to the pacemaker being inserted b. To dry up respiratory secretions to prevent aspiration during surgery c. To cause bladder relaxation during surgery so that the client will not have the urge to void during surgery d. To relax the bowel to prevent the client from having a bowel movement during surgery

b. To dry up respiratory secretions to prevent aspiration during surgery Atropine is an anticholinergic agent. The principal actions of atropine are a reduction in salivary, bronchial, and sweat gland secretions; mydriasis; cycloplegia; changes in heart rate; contraction of the bladder detrusor muscle and of the gastrointestinal smooth muscle; decreased gastric secretion; and decreased gastrointestinal motility. The reason this client is given the medication preoperatively is to dry up secretions to prevent aspiration during surgery.

A client has benign prostatic hypertrophy and has been prescribed neostigmine. The nurse is aware that this client must be monitored for what adverse effect of this medication? a. Pain with urination b. Urinary retention c. Hematuria d. Pyuria

b. Urinary retention A client with benign prostatic hypertrophy may develop urinary retention, which is an adverse effect of neostigmine.

What is typically the first observable symptom of Parkinson's disease? a. the inability to either stand erect or to rise easily from a seated position b. a resting tremor that begins in one hand, involving the fingers and thumb of one hand c. frequent falls and signs of early dementia d. the inability to either support the head or control its movement

b. a resting tremor that begins in one hand, involving the fingers and thumb of one hand

The nursing student correctly identifies the transmitter in the cholinergic neuropathways that appears insufficient in clients with Alzheimer's disease is which? a. epinephrine. b. acetylcholine. c. norepinephrine. d. memantine.

b. acetylcholine. Acetylcholine is the transmitter in the cholinergic neuropathways. Clients with early-stage Alzheimer's disease demonstrate a degeneration of these pathways. Epinephrine is a hormone and neurotransmitter; it acts on all body tissues and is responsible for such things as contraction of smooth muscles in the airways and that line the arterioles. Norepinephrine is also both a hormone and neurotransmitter; it works with epinephrine to give the body energy in times of stress. Memantine is a drug used to treat clients in late-stage Alzheimer's disease.

The nurse is caring for a client who has been diagnosed with myasthenia gravis. To ensure client safety, the nurse reviews treatment for myasthenic crisis, which requires what intervention? a. a decrease in glucocorticoid therapy b. additional anticholinesterase inhibitors c. withdrawal of anticholinesterase drugs d. withdrawal of immunoglobulin drugs

b. additional anticholinesterase inhibitors Myasthenic crisis is an exacerbation of the disease process and its clinical manifestations, requiring more anticholinesterase drug. Cholinergic crisis is a drug-induced overstimulation of the parasympathetic nervous system, requiring discontinuation of any anticholinesterase drug that the client has been receiving. Glucocorticoids and immunoglobulins are considered part of the required therapy.

A client in the emergency department has been diagnosed with mushroom poisoning. What drug does the nurse expect the health care provider to prescribe? a. pralidoxime b. atropine c. bethanechol d. neostigmine

b. atropine Accidental or intentional ingestion of certain mushrooms results in intense cholinergic stimulation and is potentially fatal. Atropine is the specific antidote for mushroom poisoning. Pralidoxime is used in the treatment of nerve gas exposure. Bethanechol is administered during the acute postoperative and postpartum periods for the treatment of nonobstructed urinary retention and neurogenic atony of the bladder muscle. Neostigmine is used in the diagnosis and treatment of myasthenia gravis

Atropine must be used cautiously in clients with a history of cardiovascular disease, due to what factor? a. atropine decreases excitability and conductivity b. atropine increases the myocardial oxygen demand c. atropine causes significant blood pressure changes d. atropine causes increased respiratory tract secretions

b. atropine increases the myocardial oxygen demand

A client, diagnosed with irritable bowel syndrome, is reporting severe abdominal pain. The nurse would anticipate an order of which drug to relieve the bowel spasms? a. trihexyphenidyl or benztropine b. dicyclomine or glycopyrrolate c. darifenacin or solifenacin d. ipraptropium or tiotropium

b. dicyclomine or glycopyrrolate

A client who is going on a cruise is concerned about motion sickness and sees the health care provider, who prescribes scopolamine. The nurse informs the client that using scopolamine may cause the client to experience: a. pupil constriction. b. drowsiness. c. diarrhea. d. urinary incontinence.

b. drowsiness. Scopolamine at therapeutic doses produces CNS depression characterized by drowsiness, euphoria, amnesia, fatigue, and dreamless sleep resulting from decreased periods of rapid eye movement.

The emergency department has been alerted to the impending arrival of several clients following possible exposure to sarin gas. The nurse knows that sarin causes harm through what process? a. distortion of the chemical structure of acetylcholine b. excessive cholinergic stimulation c. elimination of anticholinesterase d. excessive dopamine stimulation

b. excessive cholinergic stimulation Sarin is a toxic nerve gas that produces a cholinergic crisis characterized by excessive cholinergic (muscarinic) stimulation and neuromuscular blockade. This cholinergic crisis occurs because the irreversible anticholinesterase poison binds to the enzyme anticholinesterase and inactivates it. Consequently, acetylcholine remains in cholinergic synapses and causes excessive stimulation of muscarinic and nicotinic receptors. Sarin does not eliminate or distort acetylcholine nor does it cause excessive dopamine stimulation.

The client in the final stage of Alzheimer's disease tends to suffer from weight loss and eating problems. This is primarily due to: a. loss of appetite. b. inability to swallow. c. fatigue. d. inability to make food choices.

b. inability to swallow. Weight loss and eating problems related to the inability to swallow are two major problems in the late stage of AD. The other choices are factors but the inability to swallow accounts for most eating problems seen in clients with late-stage Alzheimer's disease and for most of the accompanying weight loss.

How does bethanechol affect bladder emptying? a. relaxing the urinary sphincter b. increasing the muscle tone of the detrusor muscle c. relaxing the smooth muscle that forms the urethra d. stimulating the urinary reflex in the micturition center

b. increasing the muscle tone of the detrusor muscle Bethanechol is a sympathomimetic agent that acts at the cholinergic receptors in the urinary tracts to increase muscle tone. The increased tone of the detrusor muscle in the urinary bladder allows for bladder emptying. Bethanechol does not act in the way described by any of the other options.

What are the classic symptoms associated with Parkinson's disease? Select all that apply. a. worsening eyesight b. muscle rigidity c. bradykinesia d. resting tremors e. urinary retention

b. muscle rigidity c. bradykinesia d. resting tremors

The therapeutic goals for use of anticholinergic drugs in the treatment for parkinsonism is to decrease what symptomology? a. urinary retention b. muscle spasticity c. dry mouth d. fine tremors e. salivation

b. muscle spasticity d. fine tremors e. salivation

A middle-aged client is being treated for myasthenia gravis with neostigmine. What change in the client's health status would warrant immediate medical follow-up? a. significant changes in mood and affect b. sudden muscle weakness c. migraine-type symptoms d. jaundice

b. sudden muscle weakness Sudden muscle weakness may be a sign of cholinergic crisis, which requires that the health care provider be notified immediately. The other listed changes in health also warrant assessment but are less likely to be as time dependent.

A 12 year old boy comes to the school nurse's office after falling during gym class. The nurse assesses the child and notes that he has bruising and petechiae over most of his legs, arms and torso. The child has a history of absence seizures that are treated with valproate. Based on the child's history, what would the nurse suspect? a) abuse b) leukemia c) adverse effect d) anemia

c) adverse effect (Valproic acid has an adverse effect of bleeding. The first sign of bleeding in this client is bruising and petechiae.)

The caregiver of client describes the client as having Alzheimer's disease (AD) for 10 years and is currently in the late stage of the disease. The caregiver asks the nurse if the client can go back on the medication donepezil. Which response by the nurse is appropriate? a. "Let me discuss your concerns with the prescriber." b. "A newer drug that has recently been developed is galantamine." c. "The drugs are best used early as they slow the progression of AD." d. "Donepezil has had research studies findings suggesting it cures AD."

c. "The drugs are best used early as they slow the progression of AD." Drugs used in AD early slow the mental and physical degeneration associated with the disease. The drugs must be taken routinely to slow the progression. Donepezil and galantamine are cholinesterase inhibitors. Should cholinesterase inhibitor therapy be discontinued, individuals lose any benefit they have received from the drugs within 6 weeks.

A nurse is preparing to administer a cholinergic medication. To prepare to administer the medication, the nurses is assessing the client for anticholinergic effects. Cholinergic medications act like which neurotransmitter? a. Serotonin b. Dopamine c. Acetylcholine d. Norepinephrine

c. Acetylcholine Cholinergic drugs act like the neurotransmitter acetylcholine. Serotonin, dopamine, and norepinephrine are the neurotransmitters which affect mood.

The nurse is caring for a client who is receiving an indirect-acting cholinergic medication and knows it is indicated as a treatment for myasthenia gravis as well as which other condition? a. Muscular dystrophy b. Musculoskeletal cancer c. Alzheimer's disease d. Cerebrovascular dementia

c. Alzheimer's disease Indirect-acting cholinergic or anticholinesterase drugs are indicated to treat myasthenia gravis and Alzheimer's disease.

Decreased acetylcholine levels are characteristic of what disorder? a. multiple sclerosis. b. Huntington's disease. c. Alzheimer's disease. d. Parkinson's disease.

c. Alzheimer's disease. In normal brain function, acetylcholine is an essential neurotransmitter and plays an important role in cognitive functions, including memory storage and retrieval. Alzheimer's disease, the most common type of dementia in adults, is characterized by acetylcholine abnormalities. Multiple sclerosis (MS) is a disease of the central nervous system. Protecting the nerve fibers of the central nervous system is a fatty tissue called myelin, which helps nerve fibers conduct electrical impulses. In MS, myelin is lost in multiple areas, leaving scar tissue called sclerosis. Huntington's disease is a genetic neurologic disorder characterized by abnormal body movements called chorea and a lack of coordination; it also affects a number of mental abilities and some aspects of behavior. There is no specific test or marker for Parkinson's disease (PD); diagnosis is by a health care provider and depends on the presence of at least two of the three major signs: tremor at rest, rigidity, and bradykinesia, as well as the absence of a secondary cause, such as antipsychotic medications or multiple small strokes in the regions of the brain controlling movement.

A new resident has just been admitted to the long-term care facility. When reviewing the client's medication record, the nurse observes that the client is currently taking donepezil. What should the nurse include in the resident's plan of care? a. Apply restraints when necessary during times of agitation. b. Assess the client's muscles strength and active range of motion daily. c. Assess the client's orientation each shift. d. Assess the client's respiratory status once per shift.

c. Assess the client's orientation each shift. Donepezil is used to treat Alzheimer disease. Consequently, it would be necessary to monitor the client's cognition. Respiratory and musculoskeletal status would be less affected. Restraints are only used as an absolute last resort.

A student asks the pharmacology instructor to describe the function of a cholinergic agonist. What would the instructor reply? a. Cholinergic agonists increase the activity of dopamine receptor sites throughout the brain and spinal cord. b. Cholinergic agonists decrease the activity of GABA receptor sites throughout the body. c. Cholinergic agonists increase the activity of acetylcholine receptor sites throughout the body. d. Cholinergic agonists decrease the activity of norepinephrine receptor sites throughout the brain and spinal cord.

c. Cholinergic agonists increase the activity of acetylcholine receptor sites throughout the body. Cholinergic agonists are drugs that increase the activity of acetylcholine receptor sites throughout the body. Dopamine, GABA, and norepinephrine are not associated with cholinergic agonist function.

The nurse is caring for a client with glaucoma and is aware that which category of preanesthetic medication is contraindicated in this client? a. Opioid b. Antiemetic c. Cholinergic blocking d. Antianxiety

c. Cholinergic blocking Cholinergic blocking agents are contraindicated in clients with prostatic hypertrophy, glaucoma, and myocardial ischemia. Opioids, antiemetics, and antianxiety drugs are not contraindicated in clients with glaucoma.

One of the neurotransmitters can become decreased in the area of the corpus striatum. This results in the manifestations of Parkinson's disease. Which neurotransmitter will cause this? a. Gamma-aminobutyric acid (GABA) b. Acetylcholine c. Dopamine d. Serotonin

c. Dopamine When dopamine is decreased in the area of the corpus striatum there is a chemical imbalance that allows the cholinergic or excitatory cells to dominate. This affects the functioning of the basal ganglia and cortical and cerebella components of the extrapyramidal motor system. This system provides coordination for unconscious muscle movements, including those that control position, posture, and movement. The result of the imbalance produces the signs and symptoms of Parkinson's disease. The corpus striatum in the brain is connected to the substantia nigra by a series of neurons that utilize the inhibitory neurotransmitter GABA. Higher neurons from the cerebral cortex secrete acetylcholine in the area of the corpus stratum as an excitatory neurotransmitter to coordinate movements of the body. Serotonin is not involved in these functions.

Clients with what disorder should not be given anticholinergic drugs? a. Epilepsy b. Attention deficit disorder c. Glaucoma d. Severe anxiety or hyperactivity

c. Glaucoma Anticholinergic drugs are contraindicated for patients with benign prostatic hypertrophy, myasthenia gravis, hyperthyroidism, glaucoma, tachyarrhythmia, myocardial infarction, heart failure, or conditions associated with esophageal reflux.

A female patient has been administered donepezil HCL for dementia. The patient has informed the nurse that she has also been taking nonsteroidal anti-inflammatory drugs. Which interaction should the nurse monitor for in this patient? a. Decreased effectiveness of anticholinergics b. Increased risk of theophylline toxicity c. Increased risk of GI bleeding d. Decrease GI absorption of the drug

c. Increased risk of GI bleeding The interaction of nonsteroidal anti-inflammatory drugs with cholinesterase inhibitors causes increased risk of GI bleeding, which should be monitored for. Interaction of anticholinergics with cholinesterase inhibitors causes decreased effectiveness of anticholinergics. Interaction of theophylline with cholinesterase inhibitors causes increased risk of theophylline toxicity. Interaction of nonsteroidal anti-inflammatory drugs with cholinesterase inhibitors does not decrease the GI absorption of the drug.

A 70-year-old client is taking a cholinergic blocking drug. The family has noticed the client is a confused, excited, and not voiding normally. What would be the appropriate action by the nurse in this situation? a. Instruct the client's family to continue the medication and ignore the symptoms. b. Instruct the client's family to increase the dose of the medication to combat these symptoms. c. Instruct the client's family to withhold the next dose and the nurse will contact the prescribing provider. d. Instruct the client's family to take the client to the emergency department immediately.

c. Instruct the client's family to withhold the next dose and the nurse will contact the prescribing provider. Older clients taking cholinergic blocking medications may exhibit symptoms such as excitement, mental confusion, and urinary retention. If a client is exhibiting these symptoms, the medication should be withheld for the next dose and the nurse should contact the prescribing provider. Ignoring symptoms is not advised because they could become worse. Increasing the dose would probably increase the symptoms and it is not appropriate for the nurse to change medication dosage without a provider order. It is not a medical emergency, so going to the emergency department is not necessary. The family is given the instruction due to the fact the client is confused.

A client has been prescribed an anticholinergic medication. What instructions should the nurse provide related to safety when outside in hot weather? a. Wear a hat to protect the head b. Wear sunglasses to protect the eyes c. Keep well hydrated d. Keep arms and legs covered

c. Keep well hydrated Adults taking an anticholinergic medication should be encouraged to drink plenty of fluids and to avoid hot temperatures because heat intolerance can occur and it will be important to maintain hydration should this happen. While the other options are appropriate suggestions, none are directly related to the safety issue of heat intolerance caused by the medication.

A nurse is conducting a medication resolution of a new resident of a care facility and notes that the woman has been taking neostigmine. The nurse should recognize that the woman may have a history of what health problem? a. Alzheimer disease b. Parkinson disease c. Myasthenia gravis d. Multiple sclerosis

c. Myasthenia gravis The most significant indication for neostigmine therapy is myasthenia gravis. Other clinical indications include urinary retention and paralytic ileus, but the drug is not used to treat Parkinson disease, Alzheimer disease, or multiple sclerosis.

An elderly woman is given scopolamine while on a cruise. She becomes very agitated and confused and is taken to the sick-bay of the ship. The nurse knows that which is a likely reason for these symptoms? a. Most elderly clients have dementia, which would prevent them from having normal, visual adverse reactions to cholinergic blocking drugs. b. Urinary retention often causes agitation. c. Older clients have a greater chance of adverse reactions to cholinergic blocking drugs. d. Elderly clients have more difficulty sleeping than regular clients, so the drowsiness that is a common side effect does not occur.

c. Older clients have a greater chance of adverse reactions to cholinergic blocking drugs. Older clients receiving cholinergic blocking drugs may have a greater chance of adverse reactions, including excitement, agitation, mental confusion, drowsiness, and urinary retention. Dementia does not prevent visual adverse effects. Urinary retention does not cause agitation. Drowsiness from cholinergic blocking drugs is unrelated to the sleep pattern of a client.

A patient with postoperative abdominal distention is given neostigmine for abdominal distention. What outcome will the nurse assess for? a. Absence of flatus through the rectum b. Increased urination c. Presence of bowel sounds d. Absence of bowel movements

c. Presence of bowel sounds Presence of bowel sounds would indicate increased GI muscle tone and motility and result in a therapeutic effect of the drug. Absence of flatus and a bowel movement would indicate that the medication is not producing a therapeutic effect. This drug would not cause increased urination, but could cause a decrease in urination, which would be a positive effect for neurogenic bladder.

The nurse is developing a teaching plan for an outpatient receiving cholinergic drugs. What should be the role of the nurse when developing a teaching plan? a. Suggest the patient avoid fiber-rich food during therapy. b. Suggest the patient adapt a self blood pressure measurement habit. c. Review the purpose of the drug therapy with the patient and family. d. Review the patient's previous history of disorders.

c. Review the purpose of the drug therapy with the patient and family. The nurse should review the purpose of the drug therapy with the patient and family when developing a teaching plan for an outpatient receiving cholinergic drugs. Suggesting that the patient avoid fiber-rich food during therapy, suggesting the patient adapt a self blood pressure measurement habit, and reviewing the patient's previous history of disorders are not related roles of the nurse when developing a teaching plan for an outpatient receiving cholinergic drugs.

A student asks the pharmacology instructor to explain the action of anticholinergic agents. What would be the instructor's best response? a. They block nicotinic receptors. b. They compete with serotonin for muscarinic acetylcholine receptor sites. c. They act to block the effects of the parasympathetic nervous system. d. They increase norepinephrine at the neuromuscular junction.

c. They act to block the effects of the parasympathetic nervous system. Anticholinergic drugs block the effects of the parasympathetic nervous system. Atropine is the only widely used anticholinergic drug. The drug works by blocking only the muscarinic effectors in the parasympathetic nervous system. They compete with acetylcholine for the muscarinic acetylcholine receptor sites. They do not block the nicotinic receptors and have little or no effect at the neuromuscular junction.

A client, newly diagnosed with Parkinson's disease, has been prescribed an anticholinergic drug. What assessment should the nurse perform when monitoring for adverse effects of anticholinergic agents used for the treatment of Parkinson disease? a. monitoring of laboratory results for blood dyscrasias b. focused respiratory assessment c. assessment of the client's bowel pattern d. monitoring of prothrombin time and international normalized ratio (INR)

c. assessment of the client's bowel pattern Anticholinergics often cause decreased bowel motility, resulting in constipation. This is especially true in older adults. Anticholinergics are not associated with coagulation disorders, blood dyscrasias, or respiratory problems.

Which medication is the antidote for poisoning by insecticides containing organophosphates? a. darifenacin b. solifenacin c. atropine d. benztropine

c. atropine

Which medication is the antidote for poisoning by insecticides containing organophosphates? a. darifenacin b. solifenacin c. atropine d. benztropine

c. atropine Atropine is the antidote for poisoning by cholinergic agonists such as certain species of mushrooms, cholinergic agonist drugs, cholinesterase inhibitor drugs, and insecticides containing organophosphates.

A client with diagnoses of Parkinson's disease and hepatic disease is to begin tolcapone therapy. What monitoring intervention should be included in the client's plan of care? a. daily glucose finger sticks b. annual eye examinations c. a baseline liver function test d. regular cardiograms

c. baseline liver function test

A 72-year-old man should be aware that the adverse effects of centrally acting anticholinergic drugs include: a. gout. b. diabetes mellitus. c. confusion. d. sedation.

c. confusion. Anticholinergic drugs may cause agitation, mental confusion, hallucinations, and psychosis in the older adult.

Which would lead the nurse to suspect that a client is experiencing an adverse effect to an anticholinergic agent? a. Diarrhea b. Diaphoresis c. Excess salivation d. Agitation

d. Agitation Agitation would be noted due to the blocking of central acetylcholine receptors. Constipation, reduced sweating, and dry mouth may be noted as well.

Indications for use of anticholinergic drugs in the treatment for Parkinsonism include to: a. decrease headaches. b. improve blurred vision. c. decrease salivation, spasticity, and tremors. d. decrease motor movement.

c. decrease salivation, spasticity, and tremors. Anticholinergic drugs are used in idiopathic parkinsonism to decrease salivation, spasticity, and tremors. They are used primarily in people who have minimal symptoms or who cannot tolerate levodopa, or in combination with other antiparkinson drugs.

The nurse is caring for a homebound client who is taking trospium chloride. This client likely has a history of what health problem? a. bradycardia b. irritable bowel syndrome c. urinary urgency d. Parkinson's disease

c. urinary urgency

A client's seizure activity, which is isolated to facial and neck muscles, has been identified as being caused by one area of the client's brain. The nurse should anticipate the administration of what drug? carbamazepine gabapentin clorazepate felbamate

carbamazepine Explanation: Carbamazepine is often the drug of choice for treatment of partial seizures, which are described in this scenario. It has the ability to inhibit polysynaptic responses and to block sodium channels to prevent the formation of repetitive action potentials in the abnormal focus. Clorazepate is indicated for anxiety and alcohol withdrawal and used as adjunctive therapy for partial seizures. Felbamate has been associated with severe liver failure and aplastic anemia and is now reserved for those clients who do not respond to other therapies. Gabapentin is used as adjunctive therapy in the treatment of partial seizures and for the treatment of postherpetic neuralgia.

A history of what medical condition would contraindicate the use of cyclobenzaprine for acute muscle spasms? cardiac arrhythmias diabetes mellitus transient ischemic attacks (TIA) chronic obstructive pulmonary disease (COPD)

cardiac arrhythmias Explanation: Contraindications to cyclobenzaprine include acute myocardial infarction, arrhythmia, heart block, conduction disturbances, heart failure, and hyperthyroidism. The other listed health problems do not contraindicate its use.

A client's seizure activity, which is isolated to facial and neck muscles, has been identified as being caused by one area of the client's brain. The nurse should anticipate the administration of what drug? carbamazepine gabapentin clorazepate felbamate

clorazepate Explanation: Carbamazepine is often the drug of choice for treatment of partial seizures, which are described in this scenario. It has the ability to inhibit polysynaptic responses and to block sodium channels to prevent the formation of repetitive action potentials in the abnormal focus. Clorazepate is indicated for anxiety and alcohol withdrawal and used as adjunctive therapy for partial seizures. Felbamate has been associated with severe liver failure and aplastic anemia and is now reserved for those clients who do not respond to other therapies. Gabapentin is used as adjunctive therapy in the treatment of partial seizures and for the treatment of postherpetic neuralgia.

A male client, age 75, is started on flavoxate. What adverse effect should the client be made aware of? a. Skin rash b. Urinary incontinence c. Weight gain d. Blurred vision

d. Blurred vision Some adverse effects of flavoxate include blurred vision, urinary retention, pupil dilation, photophobia, cycloplegia, increased intraocular pressure, dry mouth, constipation, and decreased sweating. Skin rash and weight gain are not noted as adverse effects of flavoxate.

The nurse is caring for a client with asthmatic bronchitis and understands that cholinergic medications are contraindicated because this classification of medication may cause which effect? a. Bronchodilation b. Decreased secretions c. Thickened secretions d. Bronchoconstriction

d. Bronchoconstriction Cholinergic drugs are contraindicated in people with asthma because they may cause bronchoconstriction and increased respiratory secretions.

A patient has been prescribed edrophonium for myasthenia gravis. Which of the following can the nurse monitor as a general adverse reaction in the patient? a. Seizure disorder b. Reduction of visual acuity c. Abdominal discomfort d. Cardiac arrhythmias

d. Cardiac arrhythmias The nurse should monitor for cardiac arrhythmias as a general adverse reaction in the patient. Seizure disorder, reduction of visual acuity, and abdominal discomfort are not edrophonium-related adverse reactions. Reduction of visual acuity is related to topical ophthalmics, and edrophonium is a muscle stimulant. When the patient is being administered bethanechol chloride for urinary retention, the nurse needs to examine abdominal discomfort as an adverse reaction of bethanechol chloride and not edrophonium.

A female client is taking cholinergic medication to control bladder retention. She presents to the emergency department with confusion, shortness of breath, and an apical pulse of 42 beats per minute and irregular. What action would the nurse expect the health care provider to take? a. Increase the cholinergic medication to increase cardiac contractility. b. Increase the cholinergic medication to increase oxygenation to the heart through vasodilation. c. Decrease the cholinergic medication to alleviate the symptom of confusion. d. Discontinue the cholinergic medication secondary to the diagnosis of bradycardia.

d. Discontinue the cholinergic medication secondary to the diagnosis of bradycardia. People with coronary artery disease should not take cholinergics because they can result in bradycardia, vasodilation, and hypotension.

Which agent would be most appropriate to administer to a client with Alzheimer's disease? a. Pyridostigmine b. Neostigmine c. Ambenonium d. Donepezil

d. Donepezil Donepezil is an indirect-acting cholinergic agent used to treat Alzheimer's disease. Pyridostigmine, neostigmine, and ambenonium would be used to treat myasthenia gravis.

The nurse is aware that cholinergic blocking agents will be used preoperatively for what type of anesthesia? a. Topical b. Local c. Regional d. General

d. General Cholinergic blocking agents are used to decrease respiratory secretions that pool in the lungs when the client's cough and swallowing reflexes are lost during general anesthesia. It would not be used prior to other forms of anesthesia unless there was the possibility of the loss of the swallowing reflexes.

Which agent would the nurse identify as acting specifically on the receptors in the GI tract? a. Ipratropium b. Tiotropium c. Trospium d. Hyoscyamine

d. Hyoscyamine Hyoscyamine acts more specifically on the receptors of the gastrointestinal tract. Ipratropium and tiotropium act more specifically to decreased respiratory secretions and cause bronchodilation. Trospium acts more specifically on the smooth muscle of the urinary tract.

Which of the following is an anticholinergic drug that may be used in the treatment of overactive bladder? a. Belladonna tincture b. Benztropine c. Trihexyphenidyl d. Oxybutynin

d. Oxybutynin Oxybutynin is prescribed in the treatment of overactive bladder.

The pharmacology instructor is discussing the adverse effects of anticholinergic drugs. What drug would the instructor indicate is administered to counteract the effects of atropine poisoning? a. Bethanechol b. Neostigmine c. Edrophonium d. Physostigmine

d. Physostigmine Physostigmine is a specific antidote for overdose of anticholinergics such as atropine.

A male client is prescribed an anticholinergic drug by his health care provider. He likes to hike with his grandchildren. The home care nurse instructs the client that anticholinergic medications have which effect? a. Increased sweating and the risk for heat stroke and dehydration b. Postural hypotension and increased risk for falls and from exposure to the elements c. Bradycardia in older adults, which increases the risk for falls and from exposure to the elements d. Prevents sweating and heat loss and increasing the risk of heat stroke

d. Prevents sweating and heat loss and increasing the risk of heat stroke Anticholinergic drugs are commonly used in home care with children and adults. The home care nurse may need to teach older clients or caregivers that the drugs prevent sweating and heat loss and increase risks of heat stroke if precautions to avoid overheating are not taken.

When assessing a client for possible adverse effects of direct-acting cholinergic agents, what might the nurse find? a. Tachycardia b. Hypertension c. Constipation d. Urinary urgency

d. Urinary urgency Adverse effects associated with direct-acting cholinergic agents include urinary urgency, bradycardia, hypotension, and diarrhea.

Unwanted anticholinergic effects include: a. wakefulness, mental alertness, and decreased fatigue. b. drowsiness or sleep; decreased muscle tone; and decreased ability to move. c. increasing hyperactivity, excessive talking, nervousness, and insomnia. d. constipation and dry mouth.

d. constipation and dry mouth. Unwanted anticholinergic effects include dry mouth, constipation, and mydriasis.

While preparing a client for an eye examination, the nurse explains that the eyedrops, an ophthalmic anticholinergic preparation, will cause what pupil reaction? a. brisk response b. constriction c. temporary fixation d. dilation

d. dilation

The nurse understands that older clients taking anticholinergic drugs are more likely to have all of the following symptoms EXCEPT: a. constipation b. blurred vision c. confusion d. hypertension

d. hypertension

What is the goal of carbidopa drug therapy when given in conjunction with levodopa? a. prevention of a cholinergic crisis b. reduction of the anticholinergic effects of levodopa c. prevention of extrapyramidal effects d. reduction of the necessary dose of levodopa

d. reduction of the necessary dose of levodopa

While most muscle relaxants drugs are central nervous system (CNS) depressants, which medication acts only on the muscle? tizanidine dantrolene metaxalone carisoprodol

dantrolene Explanation: Dantrolene is the only skeletal muscle relaxant that acts peripherally on the muscle itself; it inhibits the release of calcium in skeletal muscle cells, thereby decreasing the strength of muscle contraction.

General anesthetic has been administered to a surgical client and the client has begun to exhibit signs and symptoms of malignant hyperthermia. The operating room nurse should prepare to assist with the administration of: dantrolene. chlorzoxazone. orphenadrine. metaxalone.

dantrolene. Explanation: Indications for dantrolene include control of clinical spasticity resulting from upper motor neuron disorders; preoperatively to prevent or attenuate the development of malignant hyperthermia in susceptible clients; IV for management of fulminant malignant hyperthermia. The other drugs are not indicated for treatment of malignant hyperthermia.

What is a common risk for epileptic seizures during late infancy to early childhood? medication overdose fever learning disability malnutrition

fever Explanation: Epilepsy can be classified as idiopathic or attributable to a secondary cause. Secondary causes in infancy include developmental defects, metabolic disease, or birth injury. Fever is a common cause during late infancy and early childhood, and inherited forms usually begin in childhood or adolescence. Learning disability, medication overdose, and malnutrition are not risk factors.

What is a common risk for epileptic seizures during late infancy to early childhood?

fever- Epilepsy can be classified as idiopathic or attributable to a secondary cause. Secondary causes in infancy include developmental defects, metabolic disease, or birth injury. Fever is a common cause during late infancy and early childhood, and inherited forms usually begin in childhood or adolescence. Learning disability, medication overdose, and malnutrition are not risk factors.

A 32-year-old female client is taking tizanidine (Zanaflex) for spasticity related to her multiple sclerosis. The nurse will inform the client and her husband that the adverse effect that poses the greatest safety risk to the client is: constipation. dry mouth. fatigue. hypotension.

hypotension. Explanation: Tizanidine (Zanaflex) has been associated with hypotension, which could be a safety risk, especially if the client is also taking an antihypertensive drug. Constipation, dry mouth, and fatigue are common adverse effects that do not pose a safety risk.

The client in the final stage of Alzheimer's disease tends to suffer from weight loss and eating problems. This is primarily due to:

inability to swallow. Explanation: Weight loss and eating problems related to the inability to swallow are two major problems in the late stage of AD. The other choices are factors but the inability to swallow accounts for most eating problems seen in clients with late-stage Alzheimer's disease and for most of the accompanying weight loss.

A nurse is working with a 39-year-old client who is experiencing muscle spasms associated with multiple sclerosis. The client reports needing help with basic care The client's health care provider has prescribed baclofen (Lioresal). The nurse and the client agree that an appropriate goal related to helping to maintain self-worth would be to:

increase self-care in activities of daily living.

The nurse is caring for a client who has been diagnosed with myasthenia gravis and is receiving treatment with neostigmine. What specific effects does the nurse expect the drug to achieve? Select all that apply.

increased tone of gastrointestinal (GI) smooth muscle increased relaxation of sphincters increased salivary gland secretions decreased heart rate Explanation: In a client with myasthenia gravis, specific effects of neostigmine include decreasing the heart rate, increasing the tone of GI smooth muscle, and stimulating the salivary glands to increase secretions. In addition, this drug increases tone and contractility of smooth muscle (detrusor) in the urinary bladder and relaxes the sphincter and bronchial smooth muscles.

A client, newly diagnosed with a seizure disorder, has been prescribed valproic acid. When assessing for adverse effects, what assessment should the nurse prioritize? monitoring the client's liver enzyme levels assessing the client for signs of esophageal varices assessing the client's apical heart rate and rhythm performing musculoskeletal assessment

monitoring the client's liver enzyme levels Explanation: Valproic acid is associated with liver toxicity., so the nurse should periodically check the client's liver enzyme levels. Esophageal varices, cardiac damage and muscle weakness are not associated with valproic acid therapy.

A client has received an excessive dose of atracurium, a neuromuscular junction blocker. Which indirect-acting anticholinesterase medication will most likely be administered as an antidote?

pyridostigmine Explanation: Pyridostigmine is indicated as an antidote to neuromuscular junction blockers. Donepezil and rivastigmine are used to treat Alzheimer disease. Ambenonium is used to treat myasthenia gravis.

A nurse is caring for a client with trigeminal neuralgia at a health care facility. The health care provider has prescribed carbamazepine. In which case is the use of carbamazepine contraindicated in clients with: renal impairment. nervous breakdown. respiratory depression. hearing impairment.

renal impairment. Explanation: The nurse should know that carbamazepine is contraindicated among clients with renal impairment. Carbamazepine is not contraindicated in those with nervous breakdown, hearing impairment, and respiratory depression.

A 26-year-old woman has been prescribed the suspension form of phenytoin. The nurse will inform the patient that she should check capillary blood glucose levels frequently. take the drug with food. shake the phenytoin suspension before pouring and measuring the dose. follow the drug with water.

shake the phenytoin suspension before pouring and measuring the dose. Explanation: Teaching the importance of shaking the suspension form of phenytoin thoroughly before pouring and measuring the dose is a safety issue. Shaking the suspension ensures that a proper concentration of drug is administered. Because there is no indication that the client is diabetic or has gastrointestinal stress, it is not necessary to frequently check her capillary blood glucose level, follow the drug with water, or take the drug with meals.

The nurse is providing education to an adult client who has been prescribed lamotrigine for a seizure disorder. The nurse should inform the client that the medication must be stopped if what unexpected reaction develops?

skin rash

The nurse is providing education to an adult client who has been prescribed lamotrigine for a seizure disorder. The nurse should inform the client that the medication must be stopped if what unexpected reaction develops? anorexia insomnia skin rash fever

skin rash Explanation: The FDA has issued a black box warning related to the potential development of serious dermatologic reactions. It should be discontinued at the first sign of skin rash in an adult. Insomnia, anorexia, and fever aren't known adverse effects, so the nurse wouldn't mention them in the teachin

The nurse is providing education to an adult client who has been prescribed lamotrigine for a seizure disorder. The nurse should inform the client that the medication must be stopped if what unexpected reaction develops? anorexia insomnia fever skin rash

skin rash Explanation: The FDA has issued a black box warning related to the potential development of serious dermatologic reactions. It should be discontinued at the first sign of skin rash in an adult. Insomnia, anorexia, and fever aren't known adverse effects, so the nurse wouldn't mention them in the teaching.

What type of seizure activity is characterized by generalized tonic-clonic convulsions lasting for several minutes during which the client does not regain consciousness? febrile akinetic motor status epilepticus

status epilepticus Explanation: Status epilepticus is a life-threatening emergency characterized by generalized tonic-clonic convulsions lasting for several minutes or occurring at close intervals during which the client does not regain consciousness. None of the other options present with this described experience.


Ensembles d'études connexes

Lecture 12: epigenetic mechanism of gene regulation

View Set

PrepU Ch 37: Management of Patients with Musculoskeletal Trauma

View Set

Merrill's Ch. 17 - Liver & Biliary System

View Set

MARKETING COMMUNICATIONS: CHAPTER 9

View Set

Business Law 2 Ch. 20 True & False

View Set

Federal Tax Considerations for Life Insurance and Annuities

View Set

Chapter 14 - Developing and Pricing Goods and Services

View Set

ATI: Becoming a Professional Nurse: Socialization into Professional Nursing

View Set